tutorial class iii and iv 17-29/09/2012 1. first order...

107
. . . . . . . Tutorial Class III and IV 17-29/09/2012 . . . . . 1 First Order Partial Derivatives; Prepared by Dr. I.T. Leong for Tutorial 3-4 of Matb 210

Upload: others

Post on 25-Aug-2020

0 views

Category:

Documents


0 download

TRANSCRIPT

Page 1: Tutorial Class III and IV 17-29/09/2012 1. First Order ...fstitl/Calculus2012/2013-example-03.pdf · Tutorial Class III and IV 17-29/09/2012.. 1.First Order Partial Derivatives; 2.Higher

. . . . . .

.Tutorial Class III and IV 17-29/09/2012..

......

...1 First Order Partial Derivatives;

...2 Higher Order Partial Derivatives;

...3 Linear Approximation

...4 Chain Rule of Partial Differentiation

...5 Implicit Function and Its Partial Derivatives;

...6 Maximum and Minimum;

...7 Critical Points, and Gradient of Scalar Function;

...8 Directional Derivative;

...9 Application of Gradient;

...10 Weekly Quiz.

Prepared by Dr. I.T. Leong for Tutorial 3-4 of Matb 210

Page 2: Tutorial Class III and IV 17-29/09/2012 1. First Order ...fstitl/Calculus2012/2013-example-03.pdf · Tutorial Class III and IV 17-29/09/2012.. 1.First Order Partial Derivatives; 2.Higher

. . . . . .

.Tutorial Class III and IV 17-29/09/2012..

......

...1 First Order Partial Derivatives;

...2 Higher Order Partial Derivatives;

...3 Linear Approximation

...4 Chain Rule of Partial Differentiation

...5 Implicit Function and Its Partial Derivatives;

...6 Maximum and Minimum;

...7 Critical Points, and Gradient of Scalar Function;

...8 Directional Derivative;

...9 Application of Gradient;

...10 Weekly Quiz.

Prepared by Dr. I.T. Leong for Tutorial 3-4 of Matb 210

Page 3: Tutorial Class III and IV 17-29/09/2012 1. First Order ...fstitl/Calculus2012/2013-example-03.pdf · Tutorial Class III and IV 17-29/09/2012.. 1.First Order Partial Derivatives; 2.Higher

. . . . . .

.Tutorial Class III and IV 17-29/09/2012..

......

...1 First Order Partial Derivatives;

...2 Higher Order Partial Derivatives;

...3 Linear Approximation

...4 Chain Rule of Partial Differentiation

...5 Implicit Function and Its Partial Derivatives;

...6 Maximum and Minimum;

...7 Critical Points, and Gradient of Scalar Function;

...8 Directional Derivative;

...9 Application of Gradient;

...10 Weekly Quiz.

Prepared by Dr. I.T. Leong for Tutorial 3-4 of Matb 210

Page 4: Tutorial Class III and IV 17-29/09/2012 1. First Order ...fstitl/Calculus2012/2013-example-03.pdf · Tutorial Class III and IV 17-29/09/2012.. 1.First Order Partial Derivatives; 2.Higher

. . . . . .

.Tutorial Class III and IV 17-29/09/2012..

......

...1 First Order Partial Derivatives;

...2 Higher Order Partial Derivatives;

...3 Linear Approximation

...4 Chain Rule of Partial Differentiation

...5 Implicit Function and Its Partial Derivatives;

...6 Maximum and Minimum;

...7 Critical Points, and Gradient of Scalar Function;

...8 Directional Derivative;

...9 Application of Gradient;

...10 Weekly Quiz.

Prepared by Dr. I.T. Leong for Tutorial 3-4 of Matb 210

Page 5: Tutorial Class III and IV 17-29/09/2012 1. First Order ...fstitl/Calculus2012/2013-example-03.pdf · Tutorial Class III and IV 17-29/09/2012.. 1.First Order Partial Derivatives; 2.Higher

. . . . . .

.Tutorial Class III and IV 17-29/09/2012..

......

...1 First Order Partial Derivatives;

...2 Higher Order Partial Derivatives;

...3 Linear Approximation

...4 Chain Rule of Partial Differentiation

...5 Implicit Function and Its Partial Derivatives;

...6 Maximum and Minimum;

...7 Critical Points, and Gradient of Scalar Function;

...8 Directional Derivative;

...9 Application of Gradient;

...10 Weekly Quiz.

Prepared by Dr. I.T. Leong for Tutorial 3-4 of Matb 210

Page 6: Tutorial Class III and IV 17-29/09/2012 1. First Order ...fstitl/Calculus2012/2013-example-03.pdf · Tutorial Class III and IV 17-29/09/2012.. 1.First Order Partial Derivatives; 2.Higher

. . . . . .

.Tutorial Class III and IV 17-29/09/2012..

......

...1 First Order Partial Derivatives;

...2 Higher Order Partial Derivatives;

...3 Linear Approximation

...4 Chain Rule of Partial Differentiation

...5 Implicit Function and Its Partial Derivatives;

...6 Maximum and Minimum;

...7 Critical Points, and Gradient of Scalar Function;

...8 Directional Derivative;

...9 Application of Gradient;

...10 Weekly Quiz.

Prepared by Dr. I.T. Leong for Tutorial 3-4 of Matb 210

Page 7: Tutorial Class III and IV 17-29/09/2012 1. First Order ...fstitl/Calculus2012/2013-example-03.pdf · Tutorial Class III and IV 17-29/09/2012.. 1.First Order Partial Derivatives; 2.Higher

. . . . . .

.Tutorial Class III and IV 17-29/09/2012..

......

...1 First Order Partial Derivatives;

...2 Higher Order Partial Derivatives;

...3 Linear Approximation

...4 Chain Rule of Partial Differentiation

...5 Implicit Function and Its Partial Derivatives;

...6 Maximum and Minimum;

...7 Critical Points, and Gradient of Scalar Function;

...8 Directional Derivative;

...9 Application of Gradient;

...10 Weekly Quiz.

Prepared by Dr. I.T. Leong for Tutorial 3-4 of Matb 210

Page 8: Tutorial Class III and IV 17-29/09/2012 1. First Order ...fstitl/Calculus2012/2013-example-03.pdf · Tutorial Class III and IV 17-29/09/2012.. 1.First Order Partial Derivatives; 2.Higher

. . . . . .

.Tutorial Class III and IV 17-29/09/2012..

......

...1 First Order Partial Derivatives;

...2 Higher Order Partial Derivatives;

...3 Linear Approximation

...4 Chain Rule of Partial Differentiation

...5 Implicit Function and Its Partial Derivatives;

...6 Maximum and Minimum;

...7 Critical Points, and Gradient of Scalar Function;

...8 Directional Derivative;

...9 Application of Gradient;

...10 Weekly Quiz.

Prepared by Dr. I.T. Leong for Tutorial 3-4 of Matb 210

Page 9: Tutorial Class III and IV 17-29/09/2012 1. First Order ...fstitl/Calculus2012/2013-example-03.pdf · Tutorial Class III and IV 17-29/09/2012.. 1.First Order Partial Derivatives; 2.Higher

. . . . . .

.Tutorial Class III and IV 17-29/09/2012..

......

...1 First Order Partial Derivatives;

...2 Higher Order Partial Derivatives;

...3 Linear Approximation

...4 Chain Rule of Partial Differentiation

...5 Implicit Function and Its Partial Derivatives;

...6 Maximum and Minimum;

...7 Critical Points, and Gradient of Scalar Function;

...8 Directional Derivative;

...9 Application of Gradient;

...10 Weekly Quiz.

Prepared by Dr. I.T. Leong for Tutorial 3-4 of Matb 210

Page 10: Tutorial Class III and IV 17-29/09/2012 1. First Order ...fstitl/Calculus2012/2013-example-03.pdf · Tutorial Class III and IV 17-29/09/2012.. 1.First Order Partial Derivatives; 2.Higher

. . . . . .

.Tutorial Class III and IV 17-29/09/2012..

......

...1 First Order Partial Derivatives;

...2 Higher Order Partial Derivatives;

...3 Linear Approximation

...4 Chain Rule of Partial Differentiation

...5 Implicit Function and Its Partial Derivatives;

...6 Maximum and Minimum;

...7 Critical Points, and Gradient of Scalar Function;

...8 Directional Derivative;

...9 Application of Gradient;

...10 Weekly Quiz.

Prepared by Dr. I.T. Leong for Tutorial 3-4 of Matb 210

Page 11: Tutorial Class III and IV 17-29/09/2012 1. First Order ...fstitl/Calculus2012/2013-example-03.pdf · Tutorial Class III and IV 17-29/09/2012.. 1.First Order Partial Derivatives; 2.Higher

. . . . . .

.

......

Example. Show that the function u(x, y) = ex sin y andv(x, y) = ex cos y are solutions of Laplace’s equation uxx + uyy = 0.

Solution. (i) First we have ux = ex sin y, and uxx = ex sin y.

Similarly, uy = −ex cos y, and uyy = −ex sin y.Hence uxx + uyy = ex sin y − ex sin y = 0........Work it out (in 1 minute only!)

(ii) vx = ex cos y and vxx = ex cos y.Similarly, vy = −ex sin y, and uyy = −ex cos y.Hence, vxx + vyy = ex cos y − ex cos y = 0.

Prepared by Dr. I.T. Leong for Tutorial 3-4 of Matb 210

Page 12: Tutorial Class III and IV 17-29/09/2012 1. First Order ...fstitl/Calculus2012/2013-example-03.pdf · Tutorial Class III and IV 17-29/09/2012.. 1.First Order Partial Derivatives; 2.Higher

. . . . . .

.

......

Example. Show that the function u(x, y) = ex sin y andv(x, y) = ex cos y are solutions of Laplace’s equation uxx + uyy = 0.

Solution. (i) First we have ux = ex sin y, and uxx = ex sin y.Similarly, uy = −ex cos y, and uyy = −ex sin y.

Hence uxx + uyy = ex sin y − ex sin y = 0........Work it out (in 1 minute only!)

(ii) vx = ex cos y and vxx = ex cos y.Similarly, vy = −ex sin y, and uyy = −ex cos y.Hence, vxx + vyy = ex cos y − ex cos y = 0.

Prepared by Dr. I.T. Leong for Tutorial 3-4 of Matb 210

Page 13: Tutorial Class III and IV 17-29/09/2012 1. First Order ...fstitl/Calculus2012/2013-example-03.pdf · Tutorial Class III and IV 17-29/09/2012.. 1.First Order Partial Derivatives; 2.Higher

. . . . . .

.

......

Example. Show that the function u(x, y) = ex sin y andv(x, y) = ex cos y are solutions of Laplace’s equation uxx + uyy = 0.

Solution. (i) First we have ux = ex sin y, and uxx = ex sin y.Similarly, uy = −ex cos y, and uyy = −ex sin y.Hence uxx + uyy = ex sin y − ex sin y = 0.

.

......Work it out (in 1 minute only!)

(ii) vx = ex cos y and vxx = ex cos y.Similarly, vy = −ex sin y, and uyy = −ex cos y.Hence, vxx + vyy = ex cos y − ex cos y = 0.

Prepared by Dr. I.T. Leong for Tutorial 3-4 of Matb 210

Page 14: Tutorial Class III and IV 17-29/09/2012 1. First Order ...fstitl/Calculus2012/2013-example-03.pdf · Tutorial Class III and IV 17-29/09/2012.. 1.First Order Partial Derivatives; 2.Higher

. . . . . .

.

......

Example. Show that the function u(x, y) = ex sin y andv(x, y) = ex cos y are solutions of Laplace’s equation uxx + uyy = 0.

Solution. (i) First we have ux = ex sin y, and uxx = ex sin y.Similarly, uy = −ex cos y, and uyy = −ex sin y.Hence uxx + uyy = ex sin y − ex sin y = 0........Work it out (in 1 minute only!)

(ii) vx = ex cos y and vxx = ex cos y.Similarly, vy = −ex sin y, and uyy = −ex cos y.Hence, vxx + vyy = ex cos y − ex cos y = 0.

Prepared by Dr. I.T. Leong for Tutorial 3-4 of Matb 210

Page 15: Tutorial Class III and IV 17-29/09/2012 1. First Order ...fstitl/Calculus2012/2013-example-03.pdf · Tutorial Class III and IV 17-29/09/2012.. 1.First Order Partial Derivatives; 2.Higher

. . . . . .

.

......

Example. Show that the function u(x, y) = ex sin y andv(x, y) = ex cos y are solutions of Laplace’s equation uxx + uyy = 0.

Solution. (i) First we have ux = ex sin y, and uxx = ex sin y.Similarly, uy = −ex cos y, and uyy = −ex sin y.Hence uxx + uyy = ex sin y − ex sin y = 0........Work it out (in 1 minute only!)

(ii) vx = ex cos y and vxx = ex cos y.

Similarly, vy = −ex sin y, and uyy = −ex cos y.Hence, vxx + vyy = ex cos y − ex cos y = 0.

Prepared by Dr. I.T. Leong for Tutorial 3-4 of Matb 210

Page 16: Tutorial Class III and IV 17-29/09/2012 1. First Order ...fstitl/Calculus2012/2013-example-03.pdf · Tutorial Class III and IV 17-29/09/2012.. 1.First Order Partial Derivatives; 2.Higher

. . . . . .

.

......

Example. Show that the function u(x, y) = ex sin y andv(x, y) = ex cos y are solutions of Laplace’s equation uxx + uyy = 0.

Solution. (i) First we have ux = ex sin y, and uxx = ex sin y.Similarly, uy = −ex cos y, and uyy = −ex sin y.Hence uxx + uyy = ex sin y − ex sin y = 0........Work it out (in 1 minute only!)

(ii) vx = ex cos y and vxx = ex cos y.Similarly, vy = −ex sin y, and uyy = −ex cos y.

Hence, vxx + vyy = ex cos y − ex cos y = 0.

Prepared by Dr. I.T. Leong for Tutorial 3-4 of Matb 210

Page 17: Tutorial Class III and IV 17-29/09/2012 1. First Order ...fstitl/Calculus2012/2013-example-03.pdf · Tutorial Class III and IV 17-29/09/2012.. 1.First Order Partial Derivatives; 2.Higher

. . . . . .

.

......

Example. Show that the function u(x, y) = ex sin y andv(x, y) = ex cos y are solutions of Laplace’s equation uxx + uyy = 0.

Solution. (i) First we have ux = ex sin y, and uxx = ex sin y.Similarly, uy = −ex cos y, and uyy = −ex sin y.Hence uxx + uyy = ex sin y − ex sin y = 0........Work it out (in 1 minute only!)

(ii) vx = ex cos y and vxx = ex cos y.Similarly, vy = −ex sin y, and uyy = −ex cos y.Hence, vxx + vyy = ex cos y − ex cos y = 0.

Prepared by Dr. I.T. Leong for Tutorial 3-4 of Matb 210

Page 18: Tutorial Class III and IV 17-29/09/2012 1. First Order ...fstitl/Calculus2012/2013-example-03.pdf · Tutorial Class III and IV 17-29/09/2012.. 1.First Order Partial Derivatives; 2.Higher

. . . . . .

.

......

Example. Let u(x, y, z) = ln(x3 + y3 + z3 − 3xyz), prove thatux + uy + uz =

3x+y+z on the domain where all these functions are

well-defined.

Solution.

ux =1

x3 + y3 + z3 − 3xyz· ∂

∂x(x3 + y3 + z3 − 3xyz)

=3x2 − 3yz

x3 + y3 + z3 − 3xyz,

uy =3y2 − 3xz

x3 + y3 + z3 − 3xyz, uz =

3z2 − 3xyx3 + y3 + z3 − 3xyz

.

ux + uy + uz =3(x2 + y2 + z2 − xy − yz − xz)

x3 + y3 + z3 − 3xyz=

3x + y + z

.

The last equality follows from the algebraic identity:x3 + y3 + z3 − 3xyz = (x + y + z)(x2 + y2 + z2 − xy − yz − xz).

Prepared by Dr. I.T. Leong for Tutorial 3-4 of Matb 210

Page 19: Tutorial Class III and IV 17-29/09/2012 1. First Order ...fstitl/Calculus2012/2013-example-03.pdf · Tutorial Class III and IV 17-29/09/2012.. 1.First Order Partial Derivatives; 2.Higher

. . . . . .

.

......

Example. Let u(x, y, z) = ln(x3 + y3 + z3 − 3xyz), prove thatux + uy + uz =

3x+y+z on the domain where all these functions are

well-defined.

Solution.

ux =1

x3 + y3 + z3 − 3xyz· ∂

∂x(x3 + y3 + z3 − 3xyz)

=3x2 − 3yz

x3 + y3 + z3 − 3xyz,

uy =3y2 − 3xz

x3 + y3 + z3 − 3xyz, uz =

3z2 − 3xyx3 + y3 + z3 − 3xyz

.

ux + uy + uz =3(x2 + y2 + z2 − xy − yz − xz)

x3 + y3 + z3 − 3xyz=

3x + y + z

.

The last equality follows from the algebraic identity:x3 + y3 + z3 − 3xyz = (x + y + z)(x2 + y2 + z2 − xy − yz − xz).

Prepared by Dr. I.T. Leong for Tutorial 3-4 of Matb 210

Page 20: Tutorial Class III and IV 17-29/09/2012 1. First Order ...fstitl/Calculus2012/2013-example-03.pdf · Tutorial Class III and IV 17-29/09/2012.. 1.First Order Partial Derivatives; 2.Higher

. . . . . .

.

......

Example. Let u(x, y, z) = ln(x3 + y3 + z3 − 3xyz), prove thatux + uy + uz =

3x+y+z on the domain where all these functions are

well-defined.

Solution.

ux =1

x3 + y3 + z3 − 3xyz· ∂

∂x(x3 + y3 + z3 − 3xyz)

=3x2 − 3yz

x3 + y3 + z3 − 3xyz,

uy =3y2 − 3xz

x3 + y3 + z3 − 3xyz, uz =

3z2 − 3xyx3 + y3 + z3 − 3xyz

.

ux + uy + uz =3(x2 + y2 + z2 − xy − yz − xz)

x3 + y3 + z3 − 3xyz=

3x + y + z

.

The last equality follows from the algebraic identity:x3 + y3 + z3 − 3xyz = (x + y + z)(x2 + y2 + z2 − xy − yz − xz).

Prepared by Dr. I.T. Leong for Tutorial 3-4 of Matb 210

Page 21: Tutorial Class III and IV 17-29/09/2012 1. First Order ...fstitl/Calculus2012/2013-example-03.pdf · Tutorial Class III and IV 17-29/09/2012.. 1.First Order Partial Derivatives; 2.Higher

. . . . . .

.

......

Example. Let u(x, y, z) = ln(x3 + y3 + z3 − 3xyz), prove thatux + uy + uz =

3x+y+z on the domain where all these functions are

well-defined.

Solution.

ux =1

x3 + y3 + z3 − 3xyz· ∂

∂x(x3 + y3 + z3 − 3xyz)

=3x2 − 3yz

x3 + y3 + z3 − 3xyz,

uy =3y2 − 3xz

x3 + y3 + z3 − 3xyz, uz =

3z2 − 3xyx3 + y3 + z3 − 3xyz

.

ux + uy + uz =3(x2 + y2 + z2 − xy − yz − xz)

x3 + y3 + z3 − 3xyz

=3

x + y + z.

The last equality follows from the algebraic identity:x3 + y3 + z3 − 3xyz = (x + y + z)(x2 + y2 + z2 − xy − yz − xz).

Prepared by Dr. I.T. Leong for Tutorial 3-4 of Matb 210

Page 22: Tutorial Class III and IV 17-29/09/2012 1. First Order ...fstitl/Calculus2012/2013-example-03.pdf · Tutorial Class III and IV 17-29/09/2012.. 1.First Order Partial Derivatives; 2.Higher

. . . . . .

.

......

Example. Let u(x, y, z) = ln(x3 + y3 + z3 − 3xyz), prove thatux + uy + uz =

3x+y+z on the domain where all these functions are

well-defined.

Solution.

ux =1

x3 + y3 + z3 − 3xyz· ∂

∂x(x3 + y3 + z3 − 3xyz)

=3x2 − 3yz

x3 + y3 + z3 − 3xyz,

uy =3y2 − 3xz

x3 + y3 + z3 − 3xyz, uz =

3z2 − 3xyx3 + y3 + z3 − 3xyz

.

ux + uy + uz =3(x2 + y2 + z2 − xy − yz − xz)

x3 + y3 + z3 − 3xyz=

3x + y + z

.

The last equality follows from the algebraic identity:x3 + y3 + z3 − 3xyz = (x + y + z)(x2 + y2 + z2 − xy − yz − xz).

Prepared by Dr. I.T. Leong for Tutorial 3-4 of Matb 210

Page 23: Tutorial Class III and IV 17-29/09/2012 1. First Order ...fstitl/Calculus2012/2013-example-03.pdf · Tutorial Class III and IV 17-29/09/2012.. 1.First Order Partial Derivatives; 2.Higher

. . . . . .

.

......

Example. If z(x, y) = y + f (x2 − y2) where f is a differentiable functionof one variable on R, prove that y ∂z

∂x + x ∂z∂y = x.

Solution. This follows from chain rule that∂z∂x

=∂

∂x( y + f (x2 − y2) ) = f ′(x2 − y2)

∂x(x2 − y2) = 2xf ′(x2 − y2),

and similarly,∂z∂y

=∂

∂y( y+ f (x2 − y2) ) = 1+ f ′(x2 − y2)

∂y(x2 − y2) = 1− 2yf ′(x2 − y2).

Then we have

y∂z∂x

+ x∂z∂y

= y · 2xf ′(x2 − y2) + x · (1 − 2yf ′(x2 − y2)) = x.

Prepared by Dr. I.T. Leong for Tutorial 3-4 of Matb 210

Page 24: Tutorial Class III and IV 17-29/09/2012 1. First Order ...fstitl/Calculus2012/2013-example-03.pdf · Tutorial Class III and IV 17-29/09/2012.. 1.First Order Partial Derivatives; 2.Higher

. . . . . .

Remarks of determining the max/min values of f ....1 In general, we need some conditions to ensure that the

extremum values of f exist before we start to find them, such as,the region D discussed is close and bounded.For example, in R2, the region D is bounded by a simple closedcurve; in R3, this is more difficult as it is hard to have an analog ofsimple closed surface. In general, it is easy to check that a regionis bounded, but to check that a region is closed, one needs tocheck its compliment is open, which is technical in nature.

...2 Similar to the one variable case, extremum values can occur atthe boundary points or interior critical points.

Prepared by Dr. I.T. Leong for Tutorial 3-4 of Matb 210

Page 25: Tutorial Class III and IV 17-29/09/2012 1. First Order ...fstitl/Calculus2012/2013-example-03.pdf · Tutorial Class III and IV 17-29/09/2012.. 1.First Order Partial Derivatives; 2.Higher

. . . . . .

.

......

Example. Find abs. max. and min. values of f (x, y) = x2 − 2xy + 2yon the rectangle R = { (x, y) | 0 ≤ x ≤ 3, 0 ≤ y ≤ 2}.

Solution. Since f is a polynomial, it is continuous on the closed,bounded rectangle R, and hence f attains both max. and min. on D.For interior critical point of f : (0, 0) = ∇f (x, y) = (2x − 2y,−2x + 2),i.e. (x, y) = (1, 1). And f (1, 1) = 1.

For the boundary point, let A(0, 0), B(3, 0), C(3, 2) and D(0, 2).(i) On AB, f (x, 0) = x2 for 0 ≤ x ≤ 3, and

0 = f (0, 0) ≤ f (x, 0) ≤ f (3, 0) = 9.(ii) On BC, f (3, y) = 9 − 6y + 2y = 9 − 4y for 0 ≤ y ≤ 2, and

1 = f (3, 2) ≤ f (3, y) ≤ f (3, 0) = 9.(iii) On CD, we have f (x, 2) = x2 − 4x + 4 = (x − 2)2 for 0 ≤ x ≤ 3,

and 0 = f (2, 2) ≤ f (x, 2) ≤ f (3, 0) = 9.(iv) On DA, we have f (0, y) = 2y for 0 ≤ y ≤ 2, and

0 = f (0, 0) ≤ f (3, y) ≤ f (0, 2) = 4.The abs. max. of f on R is the max. value determined by the interiorcritical points or the ones in the boundary of R, which is f (3, 0) = 9.Similarly, abs. min. value of f on R is given by f (0, 0) = 0.

Prepared by Dr. I.T. Leong for Tutorial 3-4 of Matb 210

Page 26: Tutorial Class III and IV 17-29/09/2012 1. First Order ...fstitl/Calculus2012/2013-example-03.pdf · Tutorial Class III and IV 17-29/09/2012.. 1.First Order Partial Derivatives; 2.Higher

. . . . . .

.

......

Example. Find abs. max. and min. values of f (x, y) = x2 − 2xy + 2yon the rectangle R = { (x, y) | 0 ≤ x ≤ 3, 0 ≤ y ≤ 2}.

Solution. Since f is a polynomial, it is continuous on the closed,bounded rectangle R, and hence f attains both max. and min. on D.For interior critical point of f : (0, 0) = ∇f (x, y) = (2x − 2y,−2x + 2),i.e. (x, y) = (1, 1). And f (1, 1) = 1.For the boundary point, let A(0, 0), B(3, 0), C(3, 2) and D(0, 2).(i) On AB, f (x, 0) = x2 for 0 ≤ x ≤ 3, and

0 = f (0, 0) ≤ f (x, 0) ≤ f (3, 0) = 9.

(ii) On BC, f (3, y) = 9 − 6y + 2y = 9 − 4y for 0 ≤ y ≤ 2, and1 = f (3, 2) ≤ f (3, y) ≤ f (3, 0) = 9.

(iii) On CD, we have f (x, 2) = x2 − 4x + 4 = (x − 2)2 for 0 ≤ x ≤ 3,and 0 = f (2, 2) ≤ f (x, 2) ≤ f (3, 0) = 9.

(iv) On DA, we have f (0, y) = 2y for 0 ≤ y ≤ 2, and0 = f (0, 0) ≤ f (3, y) ≤ f (0, 2) = 4.

The abs. max. of f on R is the max. value determined by the interiorcritical points or the ones in the boundary of R, which is f (3, 0) = 9.Similarly, abs. min. value of f on R is given by f (0, 0) = 0.

Prepared by Dr. I.T. Leong for Tutorial 3-4 of Matb 210

Page 27: Tutorial Class III and IV 17-29/09/2012 1. First Order ...fstitl/Calculus2012/2013-example-03.pdf · Tutorial Class III and IV 17-29/09/2012.. 1.First Order Partial Derivatives; 2.Higher

. . . . . .

.

......

Example. Find abs. max. and min. values of f (x, y) = x2 − 2xy + 2yon the rectangle R = { (x, y) | 0 ≤ x ≤ 3, 0 ≤ y ≤ 2}.

Solution. Since f is a polynomial, it is continuous on the closed,bounded rectangle R, and hence f attains both max. and min. on D.For interior critical point of f : (0, 0) = ∇f (x, y) = (2x − 2y,−2x + 2),i.e. (x, y) = (1, 1). And f (1, 1) = 1.For the boundary point, let A(0, 0), B(3, 0), C(3, 2) and D(0, 2).(i) On AB, f (x, 0) = x2 for 0 ≤ x ≤ 3, and

0 = f (0, 0) ≤ f (x, 0) ≤ f (3, 0) = 9.(ii) On BC, f (3, y) = 9 − 6y + 2y = 9 − 4y for 0 ≤ y ≤ 2, and

1 = f (3, 2) ≤ f (3, y) ≤ f (3, 0) = 9.

(iii) On CD, we have f (x, 2) = x2 − 4x + 4 = (x − 2)2 for 0 ≤ x ≤ 3,and 0 = f (2, 2) ≤ f (x, 2) ≤ f (3, 0) = 9.

(iv) On DA, we have f (0, y) = 2y for 0 ≤ y ≤ 2, and0 = f (0, 0) ≤ f (3, y) ≤ f (0, 2) = 4.

The abs. max. of f on R is the max. value determined by the interiorcritical points or the ones in the boundary of R, which is f (3, 0) = 9.Similarly, abs. min. value of f on R is given by f (0, 0) = 0.

Prepared by Dr. I.T. Leong for Tutorial 3-4 of Matb 210

Page 28: Tutorial Class III and IV 17-29/09/2012 1. First Order ...fstitl/Calculus2012/2013-example-03.pdf · Tutorial Class III and IV 17-29/09/2012.. 1.First Order Partial Derivatives; 2.Higher

. . . . . .

.

......

Example. Find abs. max. and min. values of f (x, y) = x2 − 2xy + 2yon the rectangle R = { (x, y) | 0 ≤ x ≤ 3, 0 ≤ y ≤ 2}.

Solution. Since f is a polynomial, it is continuous on the closed,bounded rectangle R, and hence f attains both max. and min. on D.For interior critical point of f : (0, 0) = ∇f (x, y) = (2x − 2y,−2x + 2),i.e. (x, y) = (1, 1). And f (1, 1) = 1.For the boundary point, let A(0, 0), B(3, 0), C(3, 2) and D(0, 2).(i) On AB, f (x, 0) = x2 for 0 ≤ x ≤ 3, and

0 = f (0, 0) ≤ f (x, 0) ≤ f (3, 0) = 9.(ii) On BC, f (3, y) = 9 − 6y + 2y = 9 − 4y for 0 ≤ y ≤ 2, and

1 = f (3, 2) ≤ f (3, y) ≤ f (3, 0) = 9.(iii) On CD, we have f (x, 2) = x2 − 4x + 4 = (x − 2)2 for 0 ≤ x ≤ 3,

and 0 = f (2, 2) ≤ f (x, 2) ≤ f (3, 0) = 9.

(iv) On DA, we have f (0, y) = 2y for 0 ≤ y ≤ 2, and0 = f (0, 0) ≤ f (3, y) ≤ f (0, 2) = 4.

The abs. max. of f on R is the max. value determined by the interiorcritical points or the ones in the boundary of R, which is f (3, 0) = 9.Similarly, abs. min. value of f on R is given by f (0, 0) = 0.

Prepared by Dr. I.T. Leong for Tutorial 3-4 of Matb 210

Page 29: Tutorial Class III and IV 17-29/09/2012 1. First Order ...fstitl/Calculus2012/2013-example-03.pdf · Tutorial Class III and IV 17-29/09/2012.. 1.First Order Partial Derivatives; 2.Higher

. . . . . .

.

......

Example. Find abs. max. and min. values of f (x, y) = x2 − 2xy + 2yon the rectangle R = { (x, y) | 0 ≤ x ≤ 3, 0 ≤ y ≤ 2}.

Solution. Since f is a polynomial, it is continuous on the closed,bounded rectangle R, and hence f attains both max. and min. on D.For interior critical point of f : (0, 0) = ∇f (x, y) = (2x − 2y,−2x + 2),i.e. (x, y) = (1, 1). And f (1, 1) = 1.For the boundary point, let A(0, 0), B(3, 0), C(3, 2) and D(0, 2).(i) On AB, f (x, 0) = x2 for 0 ≤ x ≤ 3, and

0 = f (0, 0) ≤ f (x, 0) ≤ f (3, 0) = 9.(ii) On BC, f (3, y) = 9 − 6y + 2y = 9 − 4y for 0 ≤ y ≤ 2, and

1 = f (3, 2) ≤ f (3, y) ≤ f (3, 0) = 9.(iii) On CD, we have f (x, 2) = x2 − 4x + 4 = (x − 2)2 for 0 ≤ x ≤ 3,

and 0 = f (2, 2) ≤ f (x, 2) ≤ f (3, 0) = 9.(iv) On DA, we have f (0, y) = 2y for 0 ≤ y ≤ 2, and

0 = f (0, 0) ≤ f (3, y) ≤ f (0, 2) = 4.The abs. max. of f on R is the max. value determined by the interiorcritical points or the ones in the boundary of R, which is f (3, 0) = 9.Similarly, abs. min. value of f on R is given by f (0, 0) = 0.

Prepared by Dr. I.T. Leong for Tutorial 3-4 of Matb 210

Page 30: Tutorial Class III and IV 17-29/09/2012 1. First Order ...fstitl/Calculus2012/2013-example-03.pdf · Tutorial Class III and IV 17-29/09/2012.. 1.First Order Partial Derivatives; 2.Higher

. . . . . .

.

......Example. The absolute maximum of f (x, y) = 2xy on x2 + y2

4 ≤ 18 isA. 22 B. 36 C. 54 D. 18 E. 40

Solution. First we want to determine the interior critical point of f , so

(0, 0) = ∇f (x, y) = (2y, 2x) if and only if (x, y) = (0, 0).

Note that f (0, 0) = 0.

Next to check the points on the boundary of the regionR : x2 + y2

4 ≤ 18, which can be parameterized by

r(t) = (x(t), y(t)) = (3√

2 cos t, 6√

2 sin t) where 0 ≤ t ≤ 2π.

It follows that

f (r(t)) = 2(3√

2 cos t)(6√

2 sin t) = 36(2 sin t cos t) = 36 sin(2t),

which has a maximum f (r(π/4)) = 36 ≥ f (0, 0) = 0 when t = π/4.

Prepared by Dr. I.T. Leong for Tutorial 3-4 of Matb 210

Page 31: Tutorial Class III and IV 17-29/09/2012 1. First Order ...fstitl/Calculus2012/2013-example-03.pdf · Tutorial Class III and IV 17-29/09/2012.. 1.First Order Partial Derivatives; 2.Higher

. . . . . .

.

......Example. The absolute maximum of f (x, y) = 2xy on x2 + y2

4 ≤ 18 isA. 22 B. 36 C. 54 D. 18 E. 40

Solution. First we want to determine the interior critical point of f , so

(0, 0) = ∇f (x, y) = (2y, 2x) if and only if (x, y) = (0, 0).

Note that f (0, 0) = 0.Next to check the points on the boundary of the regionR : x2 + y2

4 ≤ 18, which can be parameterized by

r(t) = (x(t), y(t)) = (3√

2 cos t, 6√

2 sin t) where 0 ≤ t ≤ 2π.

It follows that

f (r(t)) = 2(3√

2 cos t)(6√

2 sin t) = 36(2 sin t cos t) = 36 sin(2t),

which has a maximum f (r(π/4)) = 36 ≥ f (0, 0) = 0 when t = π/4.

Prepared by Dr. I.T. Leong for Tutorial 3-4 of Matb 210

Page 32: Tutorial Class III and IV 17-29/09/2012 1. First Order ...fstitl/Calculus2012/2013-example-03.pdf · Tutorial Class III and IV 17-29/09/2012.. 1.First Order Partial Derivatives; 2.Higher

. . . . . .

.

......Example. The absolute maximum of f (x, y) = 2xy on x2 + y2

4 ≤ 18 isA. 22 B. 36 C. 54 D. 18 E. 40

Solution. First we want to determine the interior critical point of f , so

(0, 0) = ∇f (x, y) = (2y, 2x) if and only if (x, y) = (0, 0).

Note that f (0, 0) = 0.Next to check the points on the boundary of the regionR : x2 + y2

4 ≤ 18, which can be parameterized by

r(t) = (x(t), y(t)) = (3√

2 cos t, 6√

2 sin t) where 0 ≤ t ≤ 2π.

It follows that

f (r(t)) = 2(3√

2 cos t)(6√

2 sin t) = 36(2 sin t cos t) = 36 sin(2t),

which has a maximum f (r(π/4)) = 36 ≥ f (0, 0) = 0 when t = π/4.

Prepared by Dr. I.T. Leong for Tutorial 3-4 of Matb 210

Page 33: Tutorial Class III and IV 17-29/09/2012 1. First Order ...fstitl/Calculus2012/2013-example-03.pdf · Tutorial Class III and IV 17-29/09/2012.. 1.First Order Partial Derivatives; 2.Higher

. . . . . .

.

......

Example. Prove that the function f (x, y) = (1 + exp y) cos x − y exp yhas infinitely many local maxima, but no local minima at all in itsdomain.

Solution. ∇f (x, y) = ( − (1 + ey) sin x, ey cos x − yey − ey )

= ( − (1 + ey) sin x, ey(cos x − 1 − y) ).

As ey > 0, ∇f (x, y) = (0, 0) if and only if (sin x, cos x − 1 − y) = (0, 0),i.e. (x, y) = (nπ, (−1)n − 1) for some integer n.A = fxx(x, y) = −(1 + ey) cos x, C = fyy(x, y) = ey(cos x − 2 − y) )and B = fxy(x, y) = −ey cos x.At (x, y) = (2nπ, 0), we have A = −2, B = 0, C = −1 andAC − B2 = 2 > 0, hence f (2nπ, 0) = 2 are all local maxima.

Moreover, at the points (x, y) = ( (2n + 1)π, 0), we haveA = 1 + exp(−2), B = 0, C = − exp(−2) andAC − B2 = − exp(−2)− exp(−4) > 0, hence f ( (2n + 1)π, 0) are notlocal minima at all.Remark. The function g(|M|) = f (π/2, |M|) = −|M| exp(|M|)approaches to −∞ as |M| → +∞, hence f (x, y) does not have globalminimum.

Prepared by Dr. I.T. Leong for Tutorial 3-4 of Matb 210

Page 34: Tutorial Class III and IV 17-29/09/2012 1. First Order ...fstitl/Calculus2012/2013-example-03.pdf · Tutorial Class III and IV 17-29/09/2012.. 1.First Order Partial Derivatives; 2.Higher

. . . . . .

.

......

Example. Prove that the function f (x, y) = (1 + exp y) cos x − y exp yhas infinitely many local maxima, but no local minima at all in itsdomain.

Solution. ∇f (x, y) = ( − (1 + ey) sin x, ey cos x − yey − ey )

= ( − (1 + ey) sin x, ey(cos x − 1 − y) ).As ey > 0, ∇f (x, y) = (0, 0) if and only if (sin x, cos x − 1 − y) = (0, 0),i.e.

(x, y) = (nπ, (−1)n − 1) for some integer n.A = fxx(x, y) = −(1 + ey) cos x, C = fyy(x, y) = ey(cos x − 2 − y) )and B = fxy(x, y) = −ey cos x.At (x, y) = (2nπ, 0), we have A = −2, B = 0, C = −1 andAC − B2 = 2 > 0, hence f (2nπ, 0) = 2 are all local maxima.

Moreover, at the points (x, y) = ( (2n + 1)π, 0), we haveA = 1 + exp(−2), B = 0, C = − exp(−2) andAC − B2 = − exp(−2)− exp(−4) > 0, hence f ( (2n + 1)π, 0) are notlocal minima at all.Remark. The function g(|M|) = f (π/2, |M|) = −|M| exp(|M|)approaches to −∞ as |M| → +∞, hence f (x, y) does not have globalminimum.

Prepared by Dr. I.T. Leong for Tutorial 3-4 of Matb 210

Page 35: Tutorial Class III and IV 17-29/09/2012 1. First Order ...fstitl/Calculus2012/2013-example-03.pdf · Tutorial Class III and IV 17-29/09/2012.. 1.First Order Partial Derivatives; 2.Higher

. . . . . .

.

......

Example. Prove that the function f (x, y) = (1 + exp y) cos x − y exp yhas infinitely many local maxima, but no local minima at all in itsdomain.

Solution. ∇f (x, y) = ( − (1 + ey) sin x, ey cos x − yey − ey )

= ( − (1 + ey) sin x, ey(cos x − 1 − y) ).As ey > 0, ∇f (x, y) = (0, 0) if and only if (sin x, cos x − 1 − y) = (0, 0),i.e. (x, y) = (nπ, (−1)n − 1) for some integer n.

A = fxx(x, y) = −(1 + ey) cos x, C = fyy(x, y) = ey(cos x − 2 − y) )and B = fxy(x, y) = −ey cos x.At (x, y) = (2nπ, 0), we have A = −2, B = 0, C = −1 andAC − B2 = 2 > 0, hence f (2nπ, 0) = 2 are all local maxima.

Moreover, at the points (x, y) = ( (2n + 1)π, 0), we haveA = 1 + exp(−2), B = 0, C = − exp(−2) andAC − B2 = − exp(−2)− exp(−4) > 0, hence f ( (2n + 1)π, 0) are notlocal minima at all.Remark. The function g(|M|) = f (π/2, |M|) = −|M| exp(|M|)approaches to −∞ as |M| → +∞, hence f (x, y) does not have globalminimum.

Prepared by Dr. I.T. Leong for Tutorial 3-4 of Matb 210

Page 36: Tutorial Class III and IV 17-29/09/2012 1. First Order ...fstitl/Calculus2012/2013-example-03.pdf · Tutorial Class III and IV 17-29/09/2012.. 1.First Order Partial Derivatives; 2.Higher

. . . . . .

.

......

Example. Prove that the function f (x, y) = (1 + exp y) cos x − y exp yhas infinitely many local maxima, but no local minima at all in itsdomain.

Solution. ∇f (x, y) = ( − (1 + ey) sin x, ey cos x − yey − ey )

= ( − (1 + ey) sin x, ey(cos x − 1 − y) ).As ey > 0, ∇f (x, y) = (0, 0) if and only if (sin x, cos x − 1 − y) = (0, 0),i.e. (x, y) = (nπ, (−1)n − 1) for some integer n.A = fxx(x, y) = −(1 + ey) cos x, C = fyy(x, y) = ey(cos x − 2 − y) )and B = fxy(x, y) = −ey cos x.

At (x, y) = (2nπ, 0), we have A = −2, B = 0, C = −1 andAC − B2 = 2 > 0, hence f (2nπ, 0) = 2 are all local maxima.

Moreover, at the points (x, y) = ( (2n + 1)π, 0), we haveA = 1 + exp(−2), B = 0, C = − exp(−2) andAC − B2 = − exp(−2)− exp(−4) > 0, hence f ( (2n + 1)π, 0) are notlocal minima at all.Remark. The function g(|M|) = f (π/2, |M|) = −|M| exp(|M|)approaches to −∞ as |M| → +∞, hence f (x, y) does not have globalminimum.

Prepared by Dr. I.T. Leong for Tutorial 3-4 of Matb 210

Page 37: Tutorial Class III and IV 17-29/09/2012 1. First Order ...fstitl/Calculus2012/2013-example-03.pdf · Tutorial Class III and IV 17-29/09/2012.. 1.First Order Partial Derivatives; 2.Higher

. . . . . .

.

......

Example. Prove that the function f (x, y) = (1 + exp y) cos x − y exp yhas infinitely many local maxima, but no local minima at all in itsdomain.

Solution. ∇f (x, y) = ( − (1 + ey) sin x, ey cos x − yey − ey )

= ( − (1 + ey) sin x, ey(cos x − 1 − y) ).As ey > 0, ∇f (x, y) = (0, 0) if and only if (sin x, cos x − 1 − y) = (0, 0),i.e. (x, y) = (nπ, (−1)n − 1) for some integer n.A = fxx(x, y) = −(1 + ey) cos x, C = fyy(x, y) = ey(cos x − 2 − y) )and B = fxy(x, y) = −ey cos x.At (x, y) = (2nπ, 0), we have A = −2, B = 0, C = −1 andAC − B2 = 2 > 0, hence f (2nπ, 0) = 2 are all local maxima.

Moreover, at the points (x, y) = ( (2n + 1)π, 0), we haveA = 1 + exp(−2), B = 0, C = − exp(−2) andAC − B2 = − exp(−2)− exp(−4) > 0, hence f ( (2n + 1)π, 0) are notlocal minima at all.Remark. The function g(|M|) = f (π/2, |M|) = −|M| exp(|M|)approaches to −∞ as |M| → +∞, hence f (x, y) does not have globalminimum.

Prepared by Dr. I.T. Leong for Tutorial 3-4 of Matb 210

Page 38: Tutorial Class III and IV 17-29/09/2012 1. First Order ...fstitl/Calculus2012/2013-example-03.pdf · Tutorial Class III and IV 17-29/09/2012.. 1.First Order Partial Derivatives; 2.Higher

. . . . . .

.

......

Example. Prove that the function f (x, y) = (1 + exp y) cos x − y exp yhas infinitely many local maxima, but no local minima at all in itsdomain.

Solution. ∇f (x, y) = ( − (1 + ey) sin x, ey cos x − yey − ey )

= ( − (1 + ey) sin x, ey(cos x − 1 − y) ).As ey > 0, ∇f (x, y) = (0, 0) if and only if (sin x, cos x − 1 − y) = (0, 0),i.e. (x, y) = (nπ, (−1)n − 1) for some integer n.A = fxx(x, y) = −(1 + ey) cos x, C = fyy(x, y) = ey(cos x − 2 − y) )and B = fxy(x, y) = −ey cos x.At (x, y) = (2nπ, 0), we have A = −2, B = 0, C = −1 andAC − B2 = 2 > 0, hence f (2nπ, 0) = 2 are all local maxima.

Moreover, at the points (x, y) = ( (2n + 1)π, 0), we haveA = 1 + exp(−2), B = 0, C = − exp(−2) andAC − B2 = − exp(−2)− exp(−4) > 0, hence f ( (2n + 1)π, 0) are notlocal minima at all.

Remark. The function g(|M|) = f (π/2, |M|) = −|M| exp(|M|)approaches to −∞ as |M| → +∞, hence f (x, y) does not have globalminimum.

Prepared by Dr. I.T. Leong for Tutorial 3-4 of Matb 210

Page 39: Tutorial Class III and IV 17-29/09/2012 1. First Order ...fstitl/Calculus2012/2013-example-03.pdf · Tutorial Class III and IV 17-29/09/2012.. 1.First Order Partial Derivatives; 2.Higher

. . . . . .

.

......

Example. Prove that the function f (x, y) = (1 + exp y) cos x − y exp yhas infinitely many local maxima, but no local minima at all in itsdomain.

Solution. ∇f (x, y) = ( − (1 + ey) sin x, ey cos x − yey − ey )

= ( − (1 + ey) sin x, ey(cos x − 1 − y) ).As ey > 0, ∇f (x, y) = (0, 0) if and only if (sin x, cos x − 1 − y) = (0, 0),i.e. (x, y) = (nπ, (−1)n − 1) for some integer n.A = fxx(x, y) = −(1 + ey) cos x, C = fyy(x, y) = ey(cos x − 2 − y) )and B = fxy(x, y) = −ey cos x.At (x, y) = (2nπ, 0), we have A = −2, B = 0, C = −1 andAC − B2 = 2 > 0, hence f (2nπ, 0) = 2 are all local maxima.

Moreover, at the points (x, y) = ( (2n + 1)π, 0), we haveA = 1 + exp(−2), B = 0, C = − exp(−2) andAC − B2 = − exp(−2)− exp(−4) > 0, hence f ( (2n + 1)π, 0) are notlocal minima at all.Remark. The function g(|M|) = f (π/2, |M|) = −|M| exp(|M|)approaches to −∞ as |M| → +∞, hence f (x, y) does not have globalminimum.

Prepared by Dr. I.T. Leong for Tutorial 3-4 of Matb 210

Page 40: Tutorial Class III and IV 17-29/09/2012 1. First Order ...fstitl/Calculus2012/2013-example-03.pdf · Tutorial Class III and IV 17-29/09/2012.. 1.First Order Partial Derivatives; 2.Higher

. . . . . .

.

......

Definition. For any differentiable f and a point P(a, b) in its domain, let

L(x, y) = f (a, b) + fx(a, b)(x − a) + fy(a, b)(y − b)

= f (a, b) +∇f (a, b) · (x − a, y − b)

be the linear approximation of f at the point P(a, b).

Remark. For any fixed P(a, b), the function L(x, y) associated to afunction f at P(a, b) has a graph z = L(x, y) which is the tangent plane..

......

Example. Approximate the number√(3.2)2 + (3.9)2 using the linear

approximation to the function f (x, y) =√

x2 + y2 at (3, 4).

Solution. ∇f (x, y) = ( x√x2+y2

, y√x2+y2

), and ∇f (3, 4) = (3/5, 4/5),

so the linear approximation of f (x, y) at (3, 4) is given byL(x, y) = f (3, 4) +∇f (3, 4) · (x − 3, y − 4)= 5 + 3

5 (x − 3) + 45 (y − 4).

Then the number√(3.2)2 + (3.9)2 can be approximated by

L(3.2, 3.9) = 5 + 35 × 0.2 − 4

5 × 0.1 = 5 + 325 − 2

25 = 5.04.

Prepared by Dr. I.T. Leong for Tutorial 3-4 of Matb 210

Page 41: Tutorial Class III and IV 17-29/09/2012 1. First Order ...fstitl/Calculus2012/2013-example-03.pdf · Tutorial Class III and IV 17-29/09/2012.. 1.First Order Partial Derivatives; 2.Higher

. . . . . .

.

......

Definition. For any differentiable f and a point P(a, b) in its domain, let

L(x, y) = f (a, b) + fx(a, b)(x − a) + fy(a, b)(y − b)

= f (a, b) +∇f (a, b) · (x − a, y − b)

be the linear approximation of f at the point P(a, b).

Remark. For any fixed P(a, b), the function L(x, y) associated to afunction f at P(a, b) has a graph z = L(x, y) which is the tangent plane.

.

......

Example. Approximate the number√(3.2)2 + (3.9)2 using the linear

approximation to the function f (x, y) =√

x2 + y2 at (3, 4).

Solution. ∇f (x, y) = ( x√x2+y2

, y√x2+y2

), and ∇f (3, 4) = (3/5, 4/5),

so the linear approximation of f (x, y) at (3, 4) is given byL(x, y) = f (3, 4) +∇f (3, 4) · (x − 3, y − 4)= 5 + 3

5 (x − 3) + 45 (y − 4).

Then the number√(3.2)2 + (3.9)2 can be approximated by

L(3.2, 3.9) = 5 + 35 × 0.2 − 4

5 × 0.1 = 5 + 325 − 2

25 = 5.04.

Prepared by Dr. I.T. Leong for Tutorial 3-4 of Matb 210

Page 42: Tutorial Class III and IV 17-29/09/2012 1. First Order ...fstitl/Calculus2012/2013-example-03.pdf · Tutorial Class III and IV 17-29/09/2012.. 1.First Order Partial Derivatives; 2.Higher

. . . . . .

.

......

Definition. For any differentiable f and a point P(a, b) in its domain, let

L(x, y) = f (a, b) + fx(a, b)(x − a) + fy(a, b)(y − b)

= f (a, b) +∇f (a, b) · (x − a, y − b)

be the linear approximation of f at the point P(a, b).

Remark. For any fixed P(a, b), the function L(x, y) associated to afunction f at P(a, b) has a graph z = L(x, y) which is the tangent plane..

......

Example. Approximate the number√(3.2)2 + (3.9)2 using the linear

approximation to the function f (x, y) =√

x2 + y2 at (3, 4).

Solution. ∇f (x, y) = ( x√x2+y2

, y√x2+y2

), and ∇f (3, 4) = (3/5, 4/5),

so the linear approximation of f (x, y) at (3, 4) is given byL(x, y) = f (3, 4) +∇f (3, 4) · (x − 3, y − 4)= 5 + 3

5 (x − 3) + 45 (y − 4).

Then the number√(3.2)2 + (3.9)2 can be approximated by

L(3.2, 3.9) = 5 + 35 × 0.2 − 4

5 × 0.1 = 5 + 325 − 2

25 = 5.04.

Prepared by Dr. I.T. Leong for Tutorial 3-4 of Matb 210

Page 43: Tutorial Class III and IV 17-29/09/2012 1. First Order ...fstitl/Calculus2012/2013-example-03.pdf · Tutorial Class III and IV 17-29/09/2012.. 1.First Order Partial Derivatives; 2.Higher

. . . . . .

.

......

Definition. For any differentiable f and a point P(a, b) in its domain, let

L(x, y) = f (a, b) + fx(a, b)(x − a) + fy(a, b)(y − b)

= f (a, b) +∇f (a, b) · (x − a, y − b)

be the linear approximation of f at the point P(a, b).

Remark. For any fixed P(a, b), the function L(x, y) associated to afunction f at P(a, b) has a graph z = L(x, y) which is the tangent plane..

......

Example. Approximate the number√(3.2)2 + (3.9)2 using the linear

approximation to the function f (x, y) =√

x2 + y2 at (3, 4).

Solution. ∇f (x, y) = ( x√x2+y2

, y√x2+y2

), and ∇f (3, 4) = (3/5, 4/5),

so the linear approximation of f (x, y) at (3, 4) is given byL(x, y) = f (3, 4) +∇f (3, 4) · (x − 3, y − 4)

= 5 + 35 (x − 3) + 4

5 (y − 4).Then the number

√(3.2)2 + (3.9)2 can be approximated by

L(3.2, 3.9) = 5 + 35 × 0.2 − 4

5 × 0.1 = 5 + 325 − 2

25 = 5.04.

Prepared by Dr. I.T. Leong for Tutorial 3-4 of Matb 210

Page 44: Tutorial Class III and IV 17-29/09/2012 1. First Order ...fstitl/Calculus2012/2013-example-03.pdf · Tutorial Class III and IV 17-29/09/2012.. 1.First Order Partial Derivatives; 2.Higher

. . . . . .

.

......

Definition. For any differentiable f and a point P(a, b) in its domain, let

L(x, y) = f (a, b) + fx(a, b)(x − a) + fy(a, b)(y − b)

= f (a, b) +∇f (a, b) · (x − a, y − b)

be the linear approximation of f at the point P(a, b).

Remark. For any fixed P(a, b), the function L(x, y) associated to afunction f at P(a, b) has a graph z = L(x, y) which is the tangent plane..

......

Example. Approximate the number√(3.2)2 + (3.9)2 using the linear

approximation to the function f (x, y) =√

x2 + y2 at (3, 4).

Solution. ∇f (x, y) = ( x√x2+y2

, y√x2+y2

), and ∇f (3, 4) = (3/5, 4/5),

so the linear approximation of f (x, y) at (3, 4) is given byL(x, y) = f (3, 4) +∇f (3, 4) · (x − 3, y − 4)= 5 + 3

5 (x − 3) + 45 (y − 4).

Then the number√(3.2)2 + (3.9)2 can be approximated by

L(3.2, 3.9) = 5 + 35 × 0.2 − 4

5 × 0.1 = 5 + 325 − 2

25 = 5.04.

Prepared by Dr. I.T. Leong for Tutorial 3-4 of Matb 210

Page 45: Tutorial Class III and IV 17-29/09/2012 1. First Order ...fstitl/Calculus2012/2013-example-03.pdf · Tutorial Class III and IV 17-29/09/2012.. 1.First Order Partial Derivatives; 2.Higher

. . . . . .

.

......

Definition. For any differentiable f and a point P(a, b) in its domain, let

L(x, y) = f (a, b) + fx(a, b)(x − a) + fy(a, b)(y − b)

= f (a, b) +∇f (a, b) · (x − a, y − b)

be the linear approximation of f at the point P(a, b).

Remark. For any fixed P(a, b), the function L(x, y) associated to afunction f at P(a, b) has a graph z = L(x, y) which is the tangent plane..

......

Example. Approximate the number√(3.2)2 + (3.9)2 using the linear

approximation to the function f (x, y) =√

x2 + y2 at (3, 4).

Solution. ∇f (x, y) = ( x√x2+y2

, y√x2+y2

), and ∇f (3, 4) = (3/5, 4/5),

so the linear approximation of f (x, y) at (3, 4) is given byL(x, y) = f (3, 4) +∇f (3, 4) · (x − 3, y − 4)= 5 + 3

5 (x − 3) + 45 (y − 4).

Then the number√(3.2)2 + (3.9)2 can be approximated by

L(3.2, 3.9) = 5 + 35 × 0.2 − 4

5 × 0.1 = 5 + 325 − 2

25 = 5.04.

Prepared by Dr. I.T. Leong for Tutorial 3-4 of Matb 210

Page 46: Tutorial Class III and IV 17-29/09/2012 1. First Order ...fstitl/Calculus2012/2013-example-03.pdf · Tutorial Class III and IV 17-29/09/2012.. 1.First Order Partial Derivatives; 2.Higher

. . . . . .

.

......

Definition. In general, if f is a function defined in a domain D inRn (n = 2, 3), and P(x0) is a point in D, then the linear approximationof f at P is given by

L(x) = f (x0) +∇f (x0) · (x − x0),

= f (x0)

+ ∂f∂x1

(x0) · (x1 − a1) +∂f

∂x2(x0) · (x2 − a2) + · · ·+ ∂f

∂xn(x0) · (xn − an),

where ∇f (x0) = ( ∂f∂x1

(x0),∂f

∂x2(x0), · · · , ∂f

∂xn(x0) ), x0 = (a1, · · · , an),

and x = (x1, · · · , xn).

.

......

Example. The function f (x, y) = 6x2 − 2x3 + y3 + 3y2 has n criticalpoints, then n = .

Solution. ∇f (x, y) = (12x − 6x2, 3y2 + 6y), ∇f (x, y) = (0, 0) if andonly 0 = 6x(x + 2) and 0 = 2y(y + 2), it follows that(x, y) = (0, 0), (−2, 0), (0,−2), (−2,−2) are the critical points of f .Then n = 4.

Prepared by Dr. I.T. Leong for Tutorial 3-4 of Matb 210

Page 47: Tutorial Class III and IV 17-29/09/2012 1. First Order ...fstitl/Calculus2012/2013-example-03.pdf · Tutorial Class III and IV 17-29/09/2012.. 1.First Order Partial Derivatives; 2.Higher

. . . . . .

.

......

Definition. In general, if f is a function defined in a domain D inRn (n = 2, 3), and P(x0) is a point in D, then the linear approximationof f at P is given by

L(x) = f (x0) +∇f (x0) · (x − x0),

= f (x0)

+ ∂f∂x1

(x0) · (x1 − a1) +∂f

∂x2(x0) · (x2 − a2) + · · ·+ ∂f

∂xn(x0) · (xn − an),

where ∇f (x0) = ( ∂f∂x1

(x0),∂f

∂x2(x0), · · · , ∂f

∂xn(x0) ), x0 = (a1, · · · , an),

and x = (x1, · · · , xn).

.

......

Example. The function f (x, y) = 6x2 − 2x3 + y3 + 3y2 has n criticalpoints, then n = .

Solution. ∇f (x, y) = (12x − 6x2, 3y2 + 6y), ∇f (x, y) = (0, 0) if andonly 0 = 6x(x + 2) and 0 = 2y(y + 2), it follows that(x, y) = (0, 0), (−2, 0), (0,−2), (−2,−2) are the critical points of f .Then n = 4.

Prepared by Dr. I.T. Leong for Tutorial 3-4 of Matb 210

Page 48: Tutorial Class III and IV 17-29/09/2012 1. First Order ...fstitl/Calculus2012/2013-example-03.pdf · Tutorial Class III and IV 17-29/09/2012.. 1.First Order Partial Derivatives; 2.Higher

. . . . . .

.

......

The linear approximation of f (x, y) = x√

y at (1, 4) isA. 2 + 2x − y/4 B. 2 + 2x + y/4 C. 2 − 2x + y/4 D. 2x + y/4 − 1E. 2x − y/4 − 1

Solution. ∇f (x, y) = (√

y, x2√

y ), and ∇f (1, 4) = (2, 14 ),

hence the linear approximation of f at (1, 4) is given by

L(x, y) = f (1, 4) +∇f (1, 4) · (x − 1, y − 4)

= 2 + 2(x − 1) +14(y − 4)

= 2x +y4− 1.

Prepared by Dr. I.T. Leong for Tutorial 3-4 of Matb 210

Page 49: Tutorial Class III and IV 17-29/09/2012 1. First Order ...fstitl/Calculus2012/2013-example-03.pdf · Tutorial Class III and IV 17-29/09/2012.. 1.First Order Partial Derivatives; 2.Higher

. . . . . .

.Take-Home Quiz 3 (14 minutes)..

......

...1 Explain the steps to determine the absolute extremum values ofa (differentiable) function z = f (x, y).

...2 State the second derivative test for critical points of z = f (x, y).

...3 Find the gradient ∇f of the function f (x, y, z) = 1√3−x2−y2−z2

for

at (−1, 1,−1).

Solution. ∇f = ∂f∂x i + ∂f

∂y j + ∂f∂x k = .

...4 Determine the minimum of the function f (x, y) = x2 + y2, whereR = {(x, y) | x2 + y2 + xy ≤ 1 }.Hint: 4x2 + 4y2 + 4xy = (x + 2y)2 + 3x2. For the boundary of theregion R, let x

√3 = cos t, and x + 2y = sin t for 0 ≤ t ≤ 2π,

Prepared by Dr. I.T. Leong for Tutorial 3-4 of Matb 210

Page 50: Tutorial Class III and IV 17-29/09/2012 1. First Order ...fstitl/Calculus2012/2013-example-03.pdf · Tutorial Class III and IV 17-29/09/2012.. 1.First Order Partial Derivatives; 2.Higher

. . . . . .

.Chain Rule I..

......

If w = f (x, y, z) has continuous first-order partial derivatives and thatr(t) = (g(t), h(t), k(t)) is a differentiable curve in the domain of f , then

...1 the composite function F(t) = w ◦ r(t) = f (g(t), h(t), k(t)) is adifferentiable function of t, and

...2 its derivative is given bydFdt

=∂w∂x

· dxdt

+∂w∂y

· dydt

+∂w∂z

· dzdt

= ∇f (r(t)) · r′(t).

3

P(x¸, y¸, z¸)T

y

x

z

Remark. We omit the proof.

Prepared by Dr. I.T. Leong for Tutorial 3-4 of Matb 210

Page 51: Tutorial Class III and IV 17-29/09/2012 1. First Order ...fstitl/Calculus2012/2013-example-03.pdf · Tutorial Class III and IV 17-29/09/2012.. 1.First Order Partial Derivatives; 2.Higher

. . . . . .

.Chain Rule I..

......

If w = f (x, y, z) has continuous first-order partial derivatives and thatr(t) = (g(t), h(t), k(t)) is a differentiable curve in the domain of f , then

...1 the composite function F(t) = w ◦ r(t) = f (g(t), h(t), k(t)) is adifferentiable function of t, and

...2 its derivative is given bydFdt

=∂w∂x

· dxdt

+∂w∂y

· dydt

+∂w∂z

· dzdt

= ∇f (r(t)) · r′(t).

3

P(x¸, y¸, z¸)T

y

x

z

Remark. We omit the proof.

Prepared by Dr. I.T. Leong for Tutorial 3-4 of Matb 210

Page 52: Tutorial Class III and IV 17-29/09/2012 1. First Order ...fstitl/Calculus2012/2013-example-03.pdf · Tutorial Class III and IV 17-29/09/2012.. 1.First Order Partial Derivatives; 2.Higher

. . . . . .

.Chain Rule I..

......

If w = f (x, y, z) has continuous first-order partial derivatives and thatr(t) = (g(t), h(t), k(t)) is a differentiable curve in the domain of f , then

...1 the composite function F(t) = w ◦ r(t) = f (g(t), h(t), k(t)) is adifferentiable function of t, and

...2 its derivative is given bydFdt

=∂w∂x

· dxdt

+∂w∂y

· dydt

+∂w∂z

· dzdt

= ∇f (r(t)) · r′(t).

3

P(x¸, y¸, z¸)T

y

x

z

Remark. We omit the proof.Prepared by Dr. I.T. Leong for Tutorial 3-4 of Matb 210

Page 53: Tutorial Class III and IV 17-29/09/2012 1. First Order ...fstitl/Calculus2012/2013-example-03.pdf · Tutorial Class III and IV 17-29/09/2012.. 1.First Order Partial Derivatives; 2.Higher

. . . . . .

.

......

Suppose that x(t) = cos t, y(t) = sin t, z(t) = t andf (x, y, z) = exp(x2 + y2 + z2), find the derivative ofg(t) = f (x(t), y(t), z(t))with respect to t in terms of t only.A. 2et B. 2et2

C. 2e1+t2D. 2te1+t2

E. None of the above

Solution. Plugging the function into x, y, z components, we haveg(t) = f (x(t), y(t), z(t)) = f (sin t, cos t, t) = exp(cos2 t + sin2 t + t2) =

exp(1 + t2), so g′(t) = exp(1 + t2) ddt (1 + t2) = 2t exp(1 + t2).

Prepared by Dr. I.T. Leong for Tutorial 3-4 of Matb 210

Page 54: Tutorial Class III and IV 17-29/09/2012 1. First Order ...fstitl/Calculus2012/2013-example-03.pdf · Tutorial Class III and IV 17-29/09/2012.. 1.First Order Partial Derivatives; 2.Higher

. . . . . .

.

......

Suppose that x(t) = cos t, y(t) = sin t, z(t) = t andf (x, y, z) = exp(x2 + y2 + z2), find the derivative ofg(t) = f (x(t), y(t), z(t))with respect to t in terms of t only.A. 2et B. 2et2

C. 2e1+t2D. 2te1+t2

E. None of the above

Solution. Plugging the function into x, y, z components, we haveg(t) = f (x(t), y(t), z(t)) = f (sin t, cos t, t) = exp(cos2 t + sin2 t + t2) =

exp(1 + t2), so g′(t) = exp(1 + t2) ddt (1 + t2) = 2t exp(1 + t2).

Prepared by Dr. I.T. Leong for Tutorial 3-4 of Matb 210

Page 55: Tutorial Class III and IV 17-29/09/2012 1. First Order ...fstitl/Calculus2012/2013-example-03.pdf · Tutorial Class III and IV 17-29/09/2012.. 1.First Order Partial Derivatives; 2.Higher

. . . . . .

.

......

Theorem. Suppose that w = f (x, y, z) is a differentiable function,where x = x(u, v), y = y(u, v), z = z(u, v), where the coordinatefunctions are parameterized by differentiable functions. Then thecomposite function w(u, v) = f ( x(u, v), (u, v), (u, v) ) is adifferentiable function in u and v, such that the partial functions aregiven by

∂w∂u

=∂w∂x

· ∂x∂u

+∂w∂y

· ∂y∂u

+∂w∂z

· ∂z∂u

;

∂w∂v

=∂w∂x

· ∂x∂v

+∂w∂y

· ∂y∂v

+∂w∂z

· ∂z∂v

.

.

......

Remark. The formula stated above is very important in the theory ofsurface integral.

Prepared by Dr. I.T. Leong for Tutorial 3-4 of Matb 210

Page 56: Tutorial Class III and IV 17-29/09/2012 1. First Order ...fstitl/Calculus2012/2013-example-03.pdf · Tutorial Class III and IV 17-29/09/2012.. 1.First Order Partial Derivatives; 2.Higher

. . . . . .

.Chain Rules of Composite Functions..

......

Theorem. Suppose that s = f (x, y, z) is a differentiable function,where x = x(u, v, w), y = y(u, v, w), z = z(u, v, w) are thecoordinate functions parameterized by differentiable functions invariables u, v and w. Then the composite functionS(u, v, w) = f ( x(u, v, w), (u, v, w), (u, v, w) ) is differentiable in u, vand w, such that the partial functions are given by

∂S∂u

=∂S∂x

· ∂x∂u

+∂S∂y

· ∂y∂u

+∂S∂z

· ∂z∂u

;

∂S∂v

=∂S∂x

· ∂x∂v

+∂S∂y

· ∂y∂v

+∂S∂z

· ∂z∂v

;

∂S∂w

=∂S∂x

· ∂x∂w

+∂S∂y

· ∂y∂w

+∂S∂z

· ∂z∂w

.

.

......

Remark. The formula stated above is very important in the theory ofinverse function theory and integration theory.

Prepared by Dr. I.T. Leong for Tutorial 3-4 of Matb 210

Page 57: Tutorial Class III and IV 17-29/09/2012 1. First Order ...fstitl/Calculus2012/2013-example-03.pdf · Tutorial Class III and IV 17-29/09/2012.. 1.First Order Partial Derivatives; 2.Higher

. . . . . .

.

......

Example. (a) Let f (x, y) = (ex+y, ex−y). Let r(t) be a path withr(0) = (0, 0) and r′(0) = (1, 1). What is the tangent vector to theimage of r(t) under f at t = 0?

Solution. Let r(t) = (x(t), y(t)). It follows from the given condition thatx(0) = y(0) = 0, and x′(0) = y′(0) = 1. By composition, we haves(t) = (u(t), v(t)) = f ◦ r(t) = (ex(t)+y(t), ex(t)−y(t)). Then it follows

from the chain rule thatddt

u(t) =ddt(ex(t)+y(t))

= ex(t)+y(t) ddt(x(t) + y(t)) = ex(t)+y(t)(x′(t) + y′(t)).

In particular,dudt

|t=0 = e0+0 · (1 + 1) = 2, similarly,dvdt

|t=0 = e0−0 · (1 − 1) = 0. Hence the curve s(t) = f ◦ r(t) has a

tangent vector s′(0) = (2, 0) at t = 0.

Prepared by Dr. I.T. Leong for Tutorial 3-4 of Matb 210

Page 58: Tutorial Class III and IV 17-29/09/2012 1. First Order ...fstitl/Calculus2012/2013-example-03.pdf · Tutorial Class III and IV 17-29/09/2012.. 1.First Order Partial Derivatives; 2.Higher

. . . . . .

.

......

Example. Let w = f (x, y), where x = x(r, θ) = r cos θ andy = y(r, θ) = r sin θ. Consider the composite functionw = f (r cos θ, r sin θ) as a function of variables r and θ.Express ∂w

∂x , ∂w∂y and ( ∂w

∂x )2 + ( ∂w

∂y )2 in terms of ∂w

∂r and ∂w∂θ .

Solution. It follows from previous problem that1r ·

∂w∂θ = sin θ fx(r cos θ, r sin θ)− cos θ fy(r cos θ, r sin θ), and∂w∂r = cos θ fx(r cos θ, r sin θ) + sin θ fy(r cos θ, r sin θ).

Now think of a system of linear equations in unknown fx and fy:{wθr = sin θ · fx − cos θ · fy, (1)

wr = cos θ · fx + sin θ · fy. (2)Eliminate fy from (1) and (2) by considering sin θ × (1) + cos θ × (2),so

sin θ · wθr + cos θ · wr = sin2 θ fx + cos2 θ fx = fx.

Eliminate fx from (1) and (2) by considering− cos θ × (1) + sin θ × (2), so

− cos θ · wθr + sin θ · wr = cos2 θ fy + sin2 θ fy = fy, and

f 2x + f 2

y =(sin θ · wθ

r + cos θ · wr)2

+(− cos θ · wθ

r + sin θ · wr)2

= (sin2 θ + cos2 θ)

(w2

θ

r2 + w2r

)=

w2θ

r2 + w2r .Prepared by Dr. I.T. Leong for Tutorial 3-4 of Matb 210

Page 59: Tutorial Class III and IV 17-29/09/2012 1. First Order ...fstitl/Calculus2012/2013-example-03.pdf · Tutorial Class III and IV 17-29/09/2012.. 1.First Order Partial Derivatives; 2.Higher

. . . . . .

.

......

Suppose that z = f (x, y) is a function defined in a domain D, andP(a, b) is a point in D. Recall that the partial derivatives

fx(a, b) = limh→0

f (a + h, b)− f (a, b)h

, and

fy(a, b) = limk→0

f (a, b + k)− f (a, b)k

.

The limits are taken along the coordinate axes.

Prepared by Dr. I.T. Leong for Tutorial 3-4 of Matb 210

Page 60: Tutorial Class III and IV 17-29/09/2012 1. First Order ...fstitl/Calculus2012/2013-example-03.pdf · Tutorial Class III and IV 17-29/09/2012.. 1.First Order Partial Derivatives; 2.Higher

. . . . . .

.

......

Definition (Directional derivative) The resulting derivative g′(0) iscalled the directional derivative Duf of f along the direction u, andhence we write Duf = ∇f · u to represent the rate of the change of fin the unit direction u.

Remark. In general, if f = f (x1, · · · , xn) is a function of n variables,one define(i) the gradient of f to be ∇f = ( ∂f

∂x1, · · · , ∂f

∂xn), and

(ii) the directional derivative Duf by Duf = ∇f · u.

Prepared by Dr. I.T. Leong for Tutorial 3-4 of Matb 210

Page 61: Tutorial Class III and IV 17-29/09/2012 1. First Order ...fstitl/Calculus2012/2013-example-03.pdf · Tutorial Class III and IV 17-29/09/2012.. 1.First Order Partial Derivatives; 2.Higher

. . . . . .

.

......

Example. Find the directional derivative of f (x, y) = tan(x + 2y) at thepoint (0, π/6) in the direction of (−3, 4).

Solution. The unit direction vector is u =(−3, 4)√33 + 42

= (−35

,45).

Moreover, ∇f (x, y) =∇(x + 2y)

cos2(x + 2y)=

i + 2jcos2(x + 2y)

, so

∇f (0, π/6) = 4(i + 2j). Hence

Duf (0, π/6) = ∇f (0, π/6) · u = (−35)× 4+ (

85)× 4 =

−12 + 325

= 4.

Remark. The direction vector u should be of unit length, i.e. ∥u∥ = 1.

Prepared by Dr. I.T. Leong for Tutorial 3-4 of Matb 210

Page 62: Tutorial Class III and IV 17-29/09/2012 1. First Order ...fstitl/Calculus2012/2013-example-03.pdf · Tutorial Class III and IV 17-29/09/2012.. 1.First Order Partial Derivatives; 2.Higher

. . . . . .

.

......

Proposition. The greatest rate of change of a scalar function f , i.e.,the maximum directional derivative, takes place in the direction of,and has the magnitude of, the vector ∇f .

Proof. For any direction v, the directional derivative of f along thedirection v at a point P in the domain of f , is given byDv(P) := ⟨∇f (P), v

∥v∥ ⟩ = ∥∇f∥ cos θ, where θ is the angle betweenthe vectors ∇f (P) and v. Hence Dv(P) attains maximum (minimum)value if and only if cos θ = 1 (−1), if and only if ∇f (P) ( −∇f (P) ) isparallel to v. In this case, we have Dv(P) = ∥∇f∥ ( −∥∇f∥ ).

Prepared by Dr. I.T. Leong for Tutorial 3-4 of Matb 210

Page 63: Tutorial Class III and IV 17-29/09/2012 1. First Order ...fstitl/Calculus2012/2013-example-03.pdf · Tutorial Class III and IV 17-29/09/2012.. 1.First Order Partial Derivatives; 2.Higher

. . . . . .

.

......

Example. (a) Find the directional derivative of f (x, y, z) = 2x3y − 3y2zat P(1, 2,−1) in a direction v = 2i − 3j + 6k.(b) In what direction from P is the directional derivative a maximum?(c) What is the magnitude of the maximum directional derivative?

Solution.(a) ∇f (P) = 6x2yi + (2x3 − 6yz)j − 3y2k|(1,2,−1) = 12i + 14j − 12k at

P. Then the directional derivative of f along the direction v isgiven by Dvf = ∇f · v

∥v∥ = ⟨12i + 14j − 12k, 2i−3j+6k√22+32+62 ⟩ = − 90

7 .(b) Dvf (P) is maximum(minimum) ⇐⇒ v (−v) is parallel to

∇f (P) = 12i + 14j − 12k.(c) The maximum magnitude of Dvf (P) is given by

∇f · ∇f (P)∥∇f (P)∥ = ∥∇f (P)∥2

∥∇f (P)∥ = ∥∇f (P)∥ = ∥12i + 14j − 12k∥ =√

144 + 196 + 144 = 22.

Prepared by Dr. I.T. Leong for Tutorial 3-4 of Matb 210

Page 64: Tutorial Class III and IV 17-29/09/2012 1. First Order ...fstitl/Calculus2012/2013-example-03.pdf · Tutorial Class III and IV 17-29/09/2012.. 1.First Order Partial Derivatives; 2.Higher

. . . . . .

.

......

Example. The temperature in the vicinity of a polar bear is given byT(x, y, z) = e−x + e−2y + e3z. If it is at (1, 1, 1), in what direction shouldthe bear proceed in order to cool fastest?

Solution.

In order to cool the fastest, the bear should proceed in thedirection in which T is decreasing the fastest; that is, in the direction−∇T(1, 1, 1). As∇T(x, y, z) = ∂T

∂x i + ∂T∂y j + ∂T

∂z k = (−e−x,−2e−2y, 3e3x). Thus the

required direction is −∇T(1, 1, 1) = (1/e, 2/e2,−3e3)..

......

Example. Let u = f (x, y, z) = e−x2sin(xy). In what direction from

(1, π, 0) should one proceed to increase f most rapidly?

Solution. ∇f (x, y, z) = ∂u∂x i + ∂u

∂y j + ∂u∂x k =

y cos(xy)e−z2i + x cos(xy)e−z2

j − 2z sin(xy)e−z2k, and hence

∇f (1, π, 0) = (−π,−1, 0). So the required direction is the one givenby (−π,−1, 0).

Prepared by Dr. I.T. Leong for Tutorial 3-4 of Matb 210

Page 65: Tutorial Class III and IV 17-29/09/2012 1. First Order ...fstitl/Calculus2012/2013-example-03.pdf · Tutorial Class III and IV 17-29/09/2012.. 1.First Order Partial Derivatives; 2.Higher

. . . . . .

.

......

Example. The temperature in the vicinity of a polar bear is given byT(x, y, z) = e−x + e−2y + e3z. If it is at (1, 1, 1), in what direction shouldthe bear proceed in order to cool fastest?

Solution. In order to cool the fastest, the bear should proceed in thedirection in which T is decreasing the fastest; that is, in the direction−∇T(1, 1, 1). As∇T(x, y, z) = ∂T

∂x i + ∂T∂y j + ∂T

∂z k = (−e−x,−2e−2y, 3e3x). Thus the

required direction is −∇T(1, 1, 1) = (1/e, 2/e2,−3e3).

.

......

Example. Let u = f (x, y, z) = e−x2sin(xy). In what direction from

(1, π, 0) should one proceed to increase f most rapidly?

Solution. ∇f (x, y, z) = ∂u∂x i + ∂u

∂y j + ∂u∂x k =

y cos(xy)e−z2i + x cos(xy)e−z2

j − 2z sin(xy)e−z2k, and hence

∇f (1, π, 0) = (−π,−1, 0). So the required direction is the one givenby (−π,−1, 0).

Prepared by Dr. I.T. Leong for Tutorial 3-4 of Matb 210

Page 66: Tutorial Class III and IV 17-29/09/2012 1. First Order ...fstitl/Calculus2012/2013-example-03.pdf · Tutorial Class III and IV 17-29/09/2012.. 1.First Order Partial Derivatives; 2.Higher

. . . . . .

.

......

Example. The temperature in the vicinity of a polar bear is given byT(x, y, z) = e−x + e−2y + e3z. If it is at (1, 1, 1), in what direction shouldthe bear proceed in order to cool fastest?

Solution. In order to cool the fastest, the bear should proceed in thedirection in which T is decreasing the fastest; that is, in the direction−∇T(1, 1, 1). As∇T(x, y, z) = ∂T

∂x i + ∂T∂y j + ∂T

∂z k = (−e−x,−2e−2y, 3e3x). Thus the

required direction is −∇T(1, 1, 1) = (1/e, 2/e2,−3e3)..

......

Example. Let u = f (x, y, z) = e−x2sin(xy). In what direction from

(1, π, 0) should one proceed to increase f most rapidly?

Solution.

∇f (x, y, z) = ∂u∂x i + ∂u

∂y j + ∂u∂x k =

y cos(xy)e−z2i + x cos(xy)e−z2

j − 2z sin(xy)e−z2k, and hence

∇f (1, π, 0) = (−π,−1, 0). So the required direction is the one givenby (−π,−1, 0).

Prepared by Dr. I.T. Leong for Tutorial 3-4 of Matb 210

Page 67: Tutorial Class III and IV 17-29/09/2012 1. First Order ...fstitl/Calculus2012/2013-example-03.pdf · Tutorial Class III and IV 17-29/09/2012.. 1.First Order Partial Derivatives; 2.Higher

. . . . . .

.

......

Example. The temperature in the vicinity of a polar bear is given byT(x, y, z) = e−x + e−2y + e3z. If it is at (1, 1, 1), in what direction shouldthe bear proceed in order to cool fastest?

Solution. In order to cool the fastest, the bear should proceed in thedirection in which T is decreasing the fastest; that is, in the direction−∇T(1, 1, 1). As∇T(x, y, z) = ∂T

∂x i + ∂T∂y j + ∂T

∂z k = (−e−x,−2e−2y, 3e3x). Thus the

required direction is −∇T(1, 1, 1) = (1/e, 2/e2,−3e3)..

......

Example. Let u = f (x, y, z) = e−x2sin(xy). In what direction from

(1, π, 0) should one proceed to increase f most rapidly?

Solution. ∇f (x, y, z) = ∂u∂x i + ∂u

∂y j + ∂u∂x k =

y cos(xy)e−z2i + x cos(xy)e−z2

j − 2z sin(xy)e−z2k, and hence

∇f (1, π, 0) = (−π,−1, 0). So the required direction is the one givenby (−π,−1, 0).

Prepared by Dr. I.T. Leong for Tutorial 3-4 of Matb 210

Page 68: Tutorial Class III and IV 17-29/09/2012 1. First Order ...fstitl/Calculus2012/2013-example-03.pdf · Tutorial Class III and IV 17-29/09/2012.. 1.First Order Partial Derivatives; 2.Higher

. . . . . .

.

......

Example. Find the rate of change of f (x, y) = 2x3 − xy + xy2 at (2, 3)in the direction of 3i − 4j.A. 5 B. 10 C. 50 D. None of the above.

Solution.

Take u = 3i−4j∥3i−4j∥ = 3i/5 − 4j/5 = (3/5,−4/5). And

∇f (x, y) = (6x2 − y + y2,−x + 2xy). Then the directional derivativeDuf (2, 3) = ∇f (2, 3) · u = 10. Hence the correct answer is B.

Prepared by Dr. I.T. Leong for Tutorial 3-4 of Matb 210

Page 69: Tutorial Class III and IV 17-29/09/2012 1. First Order ...fstitl/Calculus2012/2013-example-03.pdf · Tutorial Class III and IV 17-29/09/2012.. 1.First Order Partial Derivatives; 2.Higher

. . . . . .

.

......

Example. Find the rate of change of f (x, y) = 2x3 − xy + xy2 at (2, 3)in the direction of 3i − 4j.A. 5 B. 10 C. 50 D. None of the above.

Solution. Take u = 3i−4j∥3i−4j∥ = 3i/5 − 4j/5 = (3/5,−4/5). And

∇f (x, y) = (6x2 − y + y2,−x + 2xy). Then the directional derivativeDuf (2, 3) = ∇f (2, 3) · u = 10. Hence the correct answer is B.

Prepared by Dr. I.T. Leong for Tutorial 3-4 of Matb 210

Page 70: Tutorial Class III and IV 17-29/09/2012 1. First Order ...fstitl/Calculus2012/2013-example-03.pdf · Tutorial Class III and IV 17-29/09/2012.. 1.First Order Partial Derivatives; 2.Higher

. . . . . .

.

......

Proposition. Let f (x, y, z) be a differentiable function defined in R3,and S : f (x, y, z) = c be a level surface for some constant c, i.e.S = { (x, y, z) | f (x, y, z) = c }. Suppose that P(a, b, c) be a point on Ssuch that the gradient vector ∇f (a, b, c) of f at point P(a, b, c) is notzero, then the equation of the tangent plane of S at P is given by< ∇f (a, b, c), (x − a, y − b, z − c) >= 0, i.e.

fx(a, b, c)(x − a) + fy(a, b, c)(y − b) + fz(a, b, c)(z − c) = 0.

.

......

Remark. For any given level surface S defined by a scalar function f ,the tangent plane of S at any P of S is spanned by the tangent vectorof the curve contained in S. The result above tells us that the normaldirection to the tangent plane of S at any point P of S is parallel to∇f (P).

Prepared by Dr. I.T. Leong for Tutorial 3-4 of Matb 210

Page 71: Tutorial Class III and IV 17-29/09/2012 1. First Order ...fstitl/Calculus2012/2013-example-03.pdf · Tutorial Class III and IV 17-29/09/2012.. 1.First Order Partial Derivatives; 2.Higher

. . . . . .

.

......

Example. Let n be the number points on the level surface (ellipsoid)f (x, y, z) = x2 + 2y2 + 4z2 + xy + 3yz = 1 at which the tangent plane isparallel to the xz-plane. Then n =

A. 0 B. 1 C. 2 D. None of the above.

Solution.

We first evaluate the gradient of f , then∇f (x, y, z) = ( 2x + y, 4y + x + 3z, 8z + 3y ).If the tangent plane of level surface given by f (x, y, z) = 1 is parallel tothe xz-plane, then ∇f is parallel to (0, 1, 0). In particular, we have(x, y, z) must satisfies y = 2x, 3y = 8z and the defining equationx2 + 2y2 + 4z2 + xy + 3yz = 1. After multiplying the last equation by16, we have 32 = 4(2x)2 + 32y2 + (8z)2 + 8(2x)y + 24(y)(8z)= 4y2 + 32y2 + (3y)2 + 8y2 + 72y2 = 125y2. So y = ±

√32/125, from

these we know the corresponding x and y coordinates.In particular, there are only two points on the level surface satisfyingthe required condition.

Prepared by Dr. I.T. Leong for Tutorial 3-4 of Matb 210

Page 72: Tutorial Class III and IV 17-29/09/2012 1. First Order ...fstitl/Calculus2012/2013-example-03.pdf · Tutorial Class III and IV 17-29/09/2012.. 1.First Order Partial Derivatives; 2.Higher

. . . . . .

.

......

Example. Let n be the number points on the level surface (ellipsoid)f (x, y, z) = x2 + 2y2 + 4z2 + xy + 3yz = 1 at which the tangent plane isparallel to the xz-plane. Then n =

A. 0 B. 1 C. 2 D. None of the above.

Solution. We first evaluate the gradient of f , then∇f (x, y, z) = ( 2x + y, 4y + x + 3z, 8z + 3y ).

If the tangent plane of level surface given by f (x, y, z) = 1 is parallel tothe xz-plane, then ∇f is parallel to (0, 1, 0). In particular, we have(x, y, z) must satisfies y = 2x, 3y = 8z and the defining equationx2 + 2y2 + 4z2 + xy + 3yz = 1. After multiplying the last equation by16, we have 32 = 4(2x)2 + 32y2 + (8z)2 + 8(2x)y + 24(y)(8z)= 4y2 + 32y2 + (3y)2 + 8y2 + 72y2 = 125y2. So y = ±

√32/125, from

these we know the corresponding x and y coordinates.In particular, there are only two points on the level surface satisfyingthe required condition.

Prepared by Dr. I.T. Leong for Tutorial 3-4 of Matb 210

Page 73: Tutorial Class III and IV 17-29/09/2012 1. First Order ...fstitl/Calculus2012/2013-example-03.pdf · Tutorial Class III and IV 17-29/09/2012.. 1.First Order Partial Derivatives; 2.Higher

. . . . . .

.

......

Example. Let n be the number points on the level surface (ellipsoid)f (x, y, z) = x2 + 2y2 + 4z2 + xy + 3yz = 1 at which the tangent plane isparallel to the xz-plane. Then n =

A. 0 B. 1 C. 2 D. None of the above.

Solution. We first evaluate the gradient of f , then∇f (x, y, z) = ( 2x + y, 4y + x + 3z, 8z + 3y ).If the tangent plane of level surface given by f (x, y, z) = 1 is parallel tothe xz-plane, then ∇f is parallel to (0, 1, 0).

In particular, we have(x, y, z) must satisfies y = 2x, 3y = 8z and the defining equationx2 + 2y2 + 4z2 + xy + 3yz = 1. After multiplying the last equation by16, we have 32 = 4(2x)2 + 32y2 + (8z)2 + 8(2x)y + 24(y)(8z)= 4y2 + 32y2 + (3y)2 + 8y2 + 72y2 = 125y2. So y = ±

√32/125, from

these we know the corresponding x and y coordinates.In particular, there are only two points on the level surface satisfyingthe required condition.

Prepared by Dr. I.T. Leong for Tutorial 3-4 of Matb 210

Page 74: Tutorial Class III and IV 17-29/09/2012 1. First Order ...fstitl/Calculus2012/2013-example-03.pdf · Tutorial Class III and IV 17-29/09/2012.. 1.First Order Partial Derivatives; 2.Higher

. . . . . .

.

......

Example. Let n be the number points on the level surface (ellipsoid)f (x, y, z) = x2 + 2y2 + 4z2 + xy + 3yz = 1 at which the tangent plane isparallel to the xz-plane. Then n =

A. 0 B. 1 C. 2 D. None of the above.

Solution. We first evaluate the gradient of f , then∇f (x, y, z) = ( 2x + y, 4y + x + 3z, 8z + 3y ).If the tangent plane of level surface given by f (x, y, z) = 1 is parallel tothe xz-plane, then ∇f is parallel to (0, 1, 0). In particular, we have(x, y, z) must satisfies y = 2x, 3y = 8z and the defining equationx2 + 2y2 + 4z2 + xy + 3yz = 1.

After multiplying the last equation by16, we have 32 = 4(2x)2 + 32y2 + (8z)2 + 8(2x)y + 24(y)(8z)= 4y2 + 32y2 + (3y)2 + 8y2 + 72y2 = 125y2. So y = ±

√32/125, from

these we know the corresponding x and y coordinates.In particular, there are only two points on the level surface satisfyingthe required condition.

Prepared by Dr. I.T. Leong for Tutorial 3-4 of Matb 210

Page 75: Tutorial Class III and IV 17-29/09/2012 1. First Order ...fstitl/Calculus2012/2013-example-03.pdf · Tutorial Class III and IV 17-29/09/2012.. 1.First Order Partial Derivatives; 2.Higher

. . . . . .

.

......

Example. Let n be the number points on the level surface (ellipsoid)f (x, y, z) = x2 + 2y2 + 4z2 + xy + 3yz = 1 at which the tangent plane isparallel to the xz-plane. Then n =

A. 0 B. 1 C. 2 D. None of the above.

Solution. We first evaluate the gradient of f , then∇f (x, y, z) = ( 2x + y, 4y + x + 3z, 8z + 3y ).If the tangent plane of level surface given by f (x, y, z) = 1 is parallel tothe xz-plane, then ∇f is parallel to (0, 1, 0). In particular, we have(x, y, z) must satisfies y = 2x, 3y = 8z and the defining equationx2 + 2y2 + 4z2 + xy + 3yz = 1. After multiplying the last equation by16, we have 32 = 4(2x)2 + 32y2 + (8z)2 + 8(2x)y + 24(y)(8z)= 4y2 + 32y2 + (3y)2 + 8y2 + 72y2 = 125y2. So y = ±

√32/125, from

these we know the corresponding x and y coordinates.

In particular, there are only two points on the level surface satisfyingthe required condition.

Prepared by Dr. I.T. Leong for Tutorial 3-4 of Matb 210

Page 76: Tutorial Class III and IV 17-29/09/2012 1. First Order ...fstitl/Calculus2012/2013-example-03.pdf · Tutorial Class III and IV 17-29/09/2012.. 1.First Order Partial Derivatives; 2.Higher

. . . . . .

.

......

Example. Let n be the number points on the level surface (ellipsoid)f (x, y, z) = x2 + 2y2 + 4z2 + xy + 3yz = 1 at which the tangent plane isparallel to the xz-plane. Then n =

A. 0 B. 1 C. 2 D. None of the above.

Solution. We first evaluate the gradient of f , then∇f (x, y, z) = ( 2x + y, 4y + x + 3z, 8z + 3y ).If the tangent plane of level surface given by f (x, y, z) = 1 is parallel tothe xz-plane, then ∇f is parallel to (0, 1, 0). In particular, we have(x, y, z) must satisfies y = 2x, 3y = 8z and the defining equationx2 + 2y2 + 4z2 + xy + 3yz = 1. After multiplying the last equation by16, we have 32 = 4(2x)2 + 32y2 + (8z)2 + 8(2x)y + 24(y)(8z)= 4y2 + 32y2 + (3y)2 + 8y2 + 72y2 = 125y2. So y = ±

√32/125, from

these we know the corresponding x and y coordinates.In particular, there are only two points on the level surface satisfyingthe required condition.

Prepared by Dr. I.T. Leong for Tutorial 3-4 of Matb 210

Page 77: Tutorial Class III and IV 17-29/09/2012 1. First Order ...fstitl/Calculus2012/2013-example-03.pdf · Tutorial Class III and IV 17-29/09/2012.. 1.First Order Partial Derivatives; 2.Higher

. . . . . .

.

......

Example. Show that the surface S : x2 − 2yz + y3 = 4 is perpendicularto any member of the family of surfaces Sa : x2 + 1 = (2 − 4a)y2 + az2

at the point of intersection P(1,−1, 2).

Solution.

Let the defining equations of level surfaces S, Sa beF(x, y, z) = x2 − 2yz + y3 − 4 = 0 andG(x, y, z) = x2 + 1 − (2 − 4a)y2 − az2 = 0. Then∇F(x, y, z) = 2xi + (3y2 − 2z)j − 2yk, and∇G(x, y, z) = 2xi − 2(2 − 4a)yj − 2azk.

Thus, the normals to the two surfaces at P(1,−1, 2) are given byN1 = 2i − j + 2k, N2 = 2i + 2(2 − 4a)j − 4ak.Since N1 · N2 = (2)(2)− 2(2 − 4a)− (2)(4a) = 0, it follows that N1and N2 are perpendicular for all a, and so the required result follows.

Prepared by Dr. I.T. Leong for Tutorial 3-4 of Matb 210

Page 78: Tutorial Class III and IV 17-29/09/2012 1. First Order ...fstitl/Calculus2012/2013-example-03.pdf · Tutorial Class III and IV 17-29/09/2012.. 1.First Order Partial Derivatives; 2.Higher

. . . . . .

.

......

Example. Show that the surface S : x2 − 2yz + y3 = 4 is perpendicularto any member of the family of surfaces Sa : x2 + 1 = (2 − 4a)y2 + az2

at the point of intersection P(1,−1, 2).

Solution. Let the defining equations of level surfaces S, Sa beF(x, y, z) = x2 − 2yz + y3 − 4 = 0 andG(x, y, z) = x2 + 1 − (2 − 4a)y2 − az2 = 0. Then∇F(x, y, z) = 2xi + (3y2 − 2z)j − 2yk, and∇G(x, y, z) = 2xi − 2(2 − 4a)yj − 2azk.

Thus, the normals to the two surfaces at P(1,−1, 2) are given byN1 = 2i − j + 2k, N2 = 2i + 2(2 − 4a)j − 4ak.Since N1 · N2 = (2)(2)− 2(2 − 4a)− (2)(4a) = 0, it follows that N1and N2 are perpendicular for all a, and so the required result follows.

Prepared by Dr. I.T. Leong for Tutorial 3-4 of Matb 210

Page 79: Tutorial Class III and IV 17-29/09/2012 1. First Order ...fstitl/Calculus2012/2013-example-03.pdf · Tutorial Class III and IV 17-29/09/2012.. 1.First Order Partial Derivatives; 2.Higher

. . . . . .

.

......

Example. Show that the surface S : x2 − 2yz + y3 = 4 is perpendicularto any member of the family of surfaces Sa : x2 + 1 = (2 − 4a)y2 + az2

at the point of intersection P(1,−1, 2).

Solution. Let the defining equations of level surfaces S, Sa beF(x, y, z) = x2 − 2yz + y3 − 4 = 0 andG(x, y, z) = x2 + 1 − (2 − 4a)y2 − az2 = 0. Then∇F(x, y, z) = 2xi + (3y2 − 2z)j − 2yk, and∇G(x, y, z) = 2xi − 2(2 − 4a)yj − 2azk.

Thus, the normals to the two surfaces at P(1,−1, 2) are given byN1 = 2i − j + 2k, N2 = 2i + 2(2 − 4a)j − 4ak.

Since N1 · N2 = (2)(2)− 2(2 − 4a)− (2)(4a) = 0, it follows that N1and N2 are perpendicular for all a, and so the required result follows.

Prepared by Dr. I.T. Leong for Tutorial 3-4 of Matb 210

Page 80: Tutorial Class III and IV 17-29/09/2012 1. First Order ...fstitl/Calculus2012/2013-example-03.pdf · Tutorial Class III and IV 17-29/09/2012.. 1.First Order Partial Derivatives; 2.Higher

. . . . . .

.

......

Example. Show that the surface S : x2 − 2yz + y3 = 4 is perpendicularto any member of the family of surfaces Sa : x2 + 1 = (2 − 4a)y2 + az2

at the point of intersection P(1,−1, 2).

Solution. Let the defining equations of level surfaces S, Sa beF(x, y, z) = x2 − 2yz + y3 − 4 = 0 andG(x, y, z) = x2 + 1 − (2 − 4a)y2 − az2 = 0. Then∇F(x, y, z) = 2xi + (3y2 − 2z)j − 2yk, and∇G(x, y, z) = 2xi − 2(2 − 4a)yj − 2azk.

Thus, the normals to the two surfaces at P(1,−1, 2) are given byN1 = 2i − j + 2k, N2 = 2i + 2(2 − 4a)j − 4ak.Since N1 · N2 = (2)(2)− 2(2 − 4a)− (2)(4a) = 0, it follows that N1and N2 are perpendicular for all a, and so the required result follows.

Prepared by Dr. I.T. Leong for Tutorial 3-4 of Matb 210

Page 81: Tutorial Class III and IV 17-29/09/2012 1. First Order ...fstitl/Calculus2012/2013-example-03.pdf · Tutorial Class III and IV 17-29/09/2012.. 1.First Order Partial Derivatives; 2.Higher

. . . . . .

.

......

Let f (x, y) be a differentiable function defined on xy-plane. For anyreal number k, recall the level level of f for k is given by the set{ (x, y) | f (x, y) = k }. When the value k changes, the level curvechanges gradually.

.

......

Let f (x, y) = x2 − 7xy + 2y2 defined onxy-plane. The blue curves represent the levelcurves Ck : f (x, y) = k of various values c. Andthe red arrows represent the gradient vectorfield ∇f (a, b) = ( fx(a, b), fy(a, b) ) which isnormal to the tangent vector to level curve Caat P(a, b) of various values k.

Prepared by Dr. I.T. Leong for Tutorial 3-4 of Matb 210

Page 82: Tutorial Class III and IV 17-29/09/2012 1. First Order ...fstitl/Calculus2012/2013-example-03.pdf · Tutorial Class III and IV 17-29/09/2012.. 1.First Order Partial Derivatives; 2.Higher

. . . . . .

.

......

Let f (x, y) be a differentiable function defined on xy-plane. For anyreal number k, recall the level level of f for k is given by the set{ (x, y) | f (x, y) = k }. When the value k changes, the level curvechanges gradually.

.

......

Let f (x, y) = x2 − 7xy + 2y2 defined onxy-plane. The blue curves represent the levelcurves Ck : f (x, y) = k of various values c. Andthe red arrows represent the gradient vectorfield ∇f (a, b) = ( fx(a, b), fy(a, b) ) which isnormal to the tangent vector to level curve Caat P(a, b) of various values k.

Prepared by Dr. I.T. Leong for Tutorial 3-4 of Matb 210

Page 83: Tutorial Class III and IV 17-29/09/2012 1. First Order ...fstitl/Calculus2012/2013-example-03.pdf · Tutorial Class III and IV 17-29/09/2012.. 1.First Order Partial Derivatives; 2.Higher

. . . . . .

.

......

Proposition. Let Ck : f (x, y) = k be a fixed level curve with a pointP(a, b) in C − k. If ∇f (a, b) ̸= (0, 0), then the equation of the tangentline of Ck at P is given by ∇f (a, b) · (x − a, y − b) = 0, i.e.fx(a, b)(x − a) + fy(a, b)(y − b) = 0.

.

......

Example. Let n be the number of points in the level curveC : 3x2 − 4y2 = −1 at which the the normal vector of C isperpendicular to the vector 4i + 3j. Then n =

A. 2 B. 1 C. 0 D. None of the above.

Solution. ∇(3x2 − 4y2) = (6x,−8y). Then ∇f ⊥ 4i + 3j if and only if24x − 24y = 0 i.e. x = y. Then −1 = 3x2 − 4x2 = −x2. Hence thereare only two points, namely (1, 1) and (−1,−1), lying on the givencurve satisfying the condition.

Prepared by Dr. I.T. Leong for Tutorial 3-4 of Matb 210

Page 84: Tutorial Class III and IV 17-29/09/2012 1. First Order ...fstitl/Calculus2012/2013-example-03.pdf · Tutorial Class III and IV 17-29/09/2012.. 1.First Order Partial Derivatives; 2.Higher

. . . . . .

.

......

Proposition. Let Ck : f (x, y) = k be a fixed level curve with a pointP(a, b) in C − k. If ∇f (a, b) ̸= (0, 0), then the equation of the tangentline of Ck at P is given by ∇f (a, b) · (x − a, y − b) = 0, i.e.fx(a, b)(x − a) + fy(a, b)(y − b) = 0.

.

......

Example. Let n be the number of points in the level curveC : 3x2 − 4y2 = −1 at which the the normal vector of C isperpendicular to the vector 4i + 3j. Then n =

A. 2 B. 1 C. 0 D. None of the above.

Solution.

∇(3x2 − 4y2) = (6x,−8y). Then ∇f ⊥ 4i + 3j if and only if24x − 24y = 0 i.e. x = y. Then −1 = 3x2 − 4x2 = −x2. Hence thereare only two points, namely (1, 1) and (−1,−1), lying on the givencurve satisfying the condition.

Prepared by Dr. I.T. Leong for Tutorial 3-4 of Matb 210

Page 85: Tutorial Class III and IV 17-29/09/2012 1. First Order ...fstitl/Calculus2012/2013-example-03.pdf · Tutorial Class III and IV 17-29/09/2012.. 1.First Order Partial Derivatives; 2.Higher

. . . . . .

.

......

Proposition. Let Ck : f (x, y) = k be a fixed level curve with a pointP(a, b) in C − k. If ∇f (a, b) ̸= (0, 0), then the equation of the tangentline of Ck at P is given by ∇f (a, b) · (x − a, y − b) = 0, i.e.fx(a, b)(x − a) + fy(a, b)(y − b) = 0.

.

......

Example. Let n be the number of points in the level curveC : 3x2 − 4y2 = −1 at which the the normal vector of C isperpendicular to the vector 4i + 3j. Then n =

A. 2 B. 1 C. 0 D. None of the above.

Solution. ∇(3x2 − 4y2) = (6x,−8y). Then ∇f ⊥ 4i + 3j if and only if24x − 24y = 0 i.e. x = y. Then −1 = 3x2 − 4x2 = −x2. Hence thereare only two points, namely (1, 1) and (−1,−1), lying on the givencurve satisfying the condition.

Prepared by Dr. I.T. Leong for Tutorial 3-4 of Matb 210

Page 86: Tutorial Class III and IV 17-29/09/2012 1. First Order ...fstitl/Calculus2012/2013-example-03.pdf · Tutorial Class III and IV 17-29/09/2012.. 1.First Order Partial Derivatives; 2.Higher

. . . . . .

.

......

Implicit Function Theorem II.If the level surface (or implicit function) S : F(x, y, z) = c defines anexplicit function z = z(x, y), and P ∈ S.

If Fz(P) ̸= 0, then∂z∂x

= −Fx

Fzand

∂z∂y

= −Fy

Fz.

.

......

Remark. In fact, Implicit Function Theorem tells us more. Iffz(a, b, c) ̸= 0, then there exists a function z = g(x, y) defined on asmall ball B(Q, δ) of Q(a, b) in xy-plane such that all the points on thelevel surface S : f (x, y, z) = f (a, b, c) is given by the graph of f , i.e.f (x, g(x, y)) = f (a, b, c). In other words, the level surface locally lookslike a deformed sheet of xy-plane. More generally, if ∇f (a, b, c) is nota zero vector, then one can choose a right projection, so that locallythe level surface S is a graph given by a differentiable function of 2variables. In particular, it is locally a smooth surface with a tangentplane. This is why level surface can be thought as an implicit function,in which one can use it to define an implicit function in which one ofthe variable is expressed in terms of the other 2 variables.

Prepared by Dr. I.T. Leong for Tutorial 3-4 of Matb 210

Page 87: Tutorial Class III and IV 17-29/09/2012 1. First Order ...fstitl/Calculus2012/2013-example-03.pdf · Tutorial Class III and IV 17-29/09/2012.. 1.First Order Partial Derivatives; 2.Higher

. . . . . .

.

......

Implicit Function Theorem II.If the level surface (or implicit function) S : F(x, y, z) = c defines anexplicit function z = z(x, y), and P ∈ S.

If Fz(P) ̸= 0, then∂z∂x

= −Fx

Fzand

∂z∂y

= −Fy

Fz.

.

......

Remark. In fact, Implicit Function Theorem tells us more. Iffz(a, b, c) ̸= 0, then there exists a function z = g(x, y) defined on asmall ball B(Q, δ) of Q(a, b) in xy-plane such that all the points on thelevel surface S : f (x, y, z) = f (a, b, c) is given by the graph of f , i.e.f (x, g(x, y)) = f (a, b, c). In other words, the level surface locally lookslike a deformed sheet of xy-plane.

More generally, if ∇f (a, b, c) is nota zero vector, then one can choose a right projection, so that locallythe level surface S is a graph given by a differentiable function of 2variables. In particular, it is locally a smooth surface with a tangentplane. This is why level surface can be thought as an implicit function,in which one can use it to define an implicit function in which one ofthe variable is expressed in terms of the other 2 variables.

Prepared by Dr. I.T. Leong for Tutorial 3-4 of Matb 210

Page 88: Tutorial Class III and IV 17-29/09/2012 1. First Order ...fstitl/Calculus2012/2013-example-03.pdf · Tutorial Class III and IV 17-29/09/2012.. 1.First Order Partial Derivatives; 2.Higher

. . . . . .

.

......

Implicit Function Theorem II.If the level surface (or implicit function) S : F(x, y, z) = c defines anexplicit function z = z(x, y), and P ∈ S.

If Fz(P) ̸= 0, then∂z∂x

= −Fx

Fzand

∂z∂y

= −Fy

Fz.

.

......

Remark. In fact, Implicit Function Theorem tells us more. Iffz(a, b, c) ̸= 0, then there exists a function z = g(x, y) defined on asmall ball B(Q, δ) of Q(a, b) in xy-plane such that all the points on thelevel surface S : f (x, y, z) = f (a, b, c) is given by the graph of f , i.e.f (x, g(x, y)) = f (a, b, c). In other words, the level surface locally lookslike a deformed sheet of xy-plane. More generally, if ∇f (a, b, c) is nota zero vector, then one can choose a right projection, so that locallythe level surface S is a graph given by a differentiable function of 2variables. In particular, it is locally a smooth surface with a tangentplane. This is why level surface can be thought as an implicit function,in which one can use it to define an implicit function in which one ofthe variable is expressed in terms of the other 2 variables.

Prepared by Dr. I.T. Leong for Tutorial 3-4 of Matb 210

Page 89: Tutorial Class III and IV 17-29/09/2012 1. First Order ...fstitl/Calculus2012/2013-example-03.pdf · Tutorial Class III and IV 17-29/09/2012.. 1.First Order Partial Derivatives; 2.Higher

. . . . . .

.

......

Example. Let z(x, y) be defined by means of an implicit functionx2 + y2 + z2 = 3xyz. Prove that ∂z

∂x = 3yz−2x2z−3xy , and ∂z

∂y = 3xz−2y2z−3xy .

Proof. Replace the variable z in the implicit function by an explicitfunction z = z(x, y), so we have x2 + y2 + (z(x, y))2 = 3xy · z(x, y) ♡in a neighborhood of the point (x, y) = (a, b) of the domain z = z(x, y).

Differentiate the identity ♡ with respect to the independent variables xand y respectively, so that we have2x + 2z(x, y) · ∂z

∂x = 3y · z(x, y) + 3xy · ∂z∂x ,

then we can collect the term ∂z∂x , so

∂z∂x (2z(x, y)− 3xy) = 3yz(x, y)− 2x, i.e. ∂z

∂x = 3yz(x,y)−2x2z(x,y)−3xy .

Remark. We usually ignore the dependent of z with respect to x andy, because it is clear that ∂z

∂x means that z is a function of on x and theremaining independent variables.We skip the details for ∂z

∂y .

Prepared by Dr. I.T. Leong for Tutorial 3-4 of Matb 210

Page 90: Tutorial Class III and IV 17-29/09/2012 1. First Order ...fstitl/Calculus2012/2013-example-03.pdf · Tutorial Class III and IV 17-29/09/2012.. 1.First Order Partial Derivatives; 2.Higher

. . . . . .

.

......

Example. Let z(x, y) be defined by means of an implicit functionx2 + y2 + z2 = 3xyz. Prove that ∂z

∂x = 3yz−2x2z−3xy , and ∂z

∂y = 3xz−2y2z−3xy .

Proof. Replace the variable z in the implicit function by an explicitfunction z = z(x, y), so we have x2 + y2 + (z(x, y))2 = 3xy · z(x, y) ♡in a neighborhood of the point (x, y) = (a, b) of the domain z = z(x, y).Differentiate the identity ♡ with respect to the independent variables xand y respectively, so that we have2x + 2z(x, y) · ∂z

∂x = 3y · z(x, y) + 3xy · ∂z∂x ,

then we can collect the term ∂z∂x , so

∂z∂x (2z(x, y)− 3xy) = 3yz(x, y)− 2x, i.e. ∂z

∂x = 3yz(x,y)−2x2z(x,y)−3xy .

Remark. We usually ignore the dependent of z with respect to x andy, because it is clear that ∂z

∂x means that z is a function of on x and theremaining independent variables.We skip the details for ∂z

∂y .

Prepared by Dr. I.T. Leong for Tutorial 3-4 of Matb 210

Page 91: Tutorial Class III and IV 17-29/09/2012 1. First Order ...fstitl/Calculus2012/2013-example-03.pdf · Tutorial Class III and IV 17-29/09/2012.. 1.First Order Partial Derivatives; 2.Higher

. . . . . .

.

......

Example. Let z(x, y) be defined by means of an implicit functionx2 + y2 + z2 = 3xyz. Prove that ∂z

∂x = 3yz−2x2z−3xy , and ∂z

∂y = 3xz−2y2z−3xy .

Proof. Replace the variable z in the implicit function by an explicitfunction z = z(x, y), so we have x2 + y2 + (z(x, y))2 = 3xy · z(x, y) ♡in a neighborhood of the point (x, y) = (a, b) of the domain z = z(x, y).Differentiate the identity ♡ with respect to the independent variables xand y respectively, so that we have2x + 2z(x, y) · ∂z

∂x = 3y · z(x, y) + 3xy · ∂z∂x ,

then we can collect the term ∂z∂x , so

∂z∂x (2z(x, y)− 3xy) = 3yz(x, y)− 2x, i.e. ∂z

∂x = 3yz(x,y)−2x2z(x,y)−3xy .

Remark. We usually ignore the dependent of z with respect to x andy, because it is clear that ∂z

∂x means that z is a function of on x and theremaining independent variables.We skip the details for ∂z

∂y .

Prepared by Dr. I.T. Leong for Tutorial 3-4 of Matb 210

Page 92: Tutorial Class III and IV 17-29/09/2012 1. First Order ...fstitl/Calculus2012/2013-example-03.pdf · Tutorial Class III and IV 17-29/09/2012.. 1.First Order Partial Derivatives; 2.Higher

. . . . . .

.

......

Example. Let z(x, y) be defined by means of an implicit functionx2 + y2 + z2 = 3xyz. Prove that ∂z

∂x = 3yz−2x2z−3xy , and ∂z

∂y = 3xz−2y2z−3xy .

Proof. Replace the variable z in the implicit function by an explicitfunction z = z(x, y), so we have x2 + y2 + (z(x, y))2 = 3xy · z(x, y) ♡in a neighborhood of the point (x, y) = (a, b) of the domain z = z(x, y).Differentiate the identity ♡ with respect to the independent variables xand y respectively, so that we have2x + 2z(x, y) · ∂z

∂x = 3y · z(x, y) + 3xy · ∂z∂x ,

then we can collect the term ∂z∂x , so

∂z∂x (2z(x, y)− 3xy) = 3yz(x, y)− 2x, i.e. ∂z

∂x = 3yz(x,y)−2x2z(x,y)−3xy .

Remark. We usually ignore the dependent of z with respect to x andy, because it is clear that ∂z

∂x means that z is a function of on x and theremaining independent variables.

We skip the details for ∂z∂y .

Prepared by Dr. I.T. Leong for Tutorial 3-4 of Matb 210

Page 93: Tutorial Class III and IV 17-29/09/2012 1. First Order ...fstitl/Calculus2012/2013-example-03.pdf · Tutorial Class III and IV 17-29/09/2012.. 1.First Order Partial Derivatives; 2.Higher

. . . . . .

.

......

Example. Let z(x, y) be defined by means of an implicit functionx2 + y2 + z2 = 3xyz. Prove that ∂z

∂x = 3yz−2x2z−3xy , and ∂z

∂y = 3xz−2y2z−3xy .

Proof. Replace the variable z in the implicit function by an explicitfunction z = z(x, y), so we have x2 + y2 + (z(x, y))2 = 3xy · z(x, y) ♡in a neighborhood of the point (x, y) = (a, b) of the domain z = z(x, y).Differentiate the identity ♡ with respect to the independent variables xand y respectively, so that we have2x + 2z(x, y) · ∂z

∂x = 3y · z(x, y) + 3xy · ∂z∂x ,

then we can collect the term ∂z∂x , so

∂z∂x (2z(x, y)− 3xy) = 3yz(x, y)− 2x, i.e. ∂z

∂x = 3yz(x,y)−2x2z(x,y)−3xy .

Remark. We usually ignore the dependent of z with respect to x andy, because it is clear that ∂z

∂x means that z is a function of on x and theremaining independent variables.We skip the details for ∂z

∂y .

Prepared by Dr. I.T. Leong for Tutorial 3-4 of Matb 210

Page 94: Tutorial Class III and IV 17-29/09/2012 1. First Order ...fstitl/Calculus2012/2013-example-03.pdf · Tutorial Class III and IV 17-29/09/2012.. 1.First Order Partial Derivatives; 2.Higher

. . . . . .

.

......

Example. Determine the extremum of the function z = z(x, y) definedimplicitly by the equation 3x2 + 2y2 + z2 + 8yz − z + 8 = 0.

Solution. Define F(x, y, z) = 3x2 + 2y2 + z2 + 8yz − z + 8, so thefunction z = z(x, y) is in fact the graph of the level surface Sassociated to the equation F(x, y, z) = 0, or sometimes we just denoteit by S : F(x, y, z) = 0.

It follows that F(x, y, z(x, y)) = 0, for all (x, y) inthe (unspecified) domain of z(x, y), in fact we just think of the equalityas an identity in x and y. So differentiate with respect to x and yrespectively by means of chain rule, we have

0 =∂

∂x(0) =

∂x( F(x, y, z(x, y)) ) =

∂F∂x

∂x∂x

+∂F∂z

∂z∂x

= Fx + Fz∂z∂x

, so

one has∂z∂x

(x, y) = −Fx(x, y, z(x, y))Fz(x, y, z(x, y))

= −Fx

Fzand

∂z∂y

(x, y) = −Fy(x, y, z(x, y))Fz(x, y, z(x, y))

= −Fy

Fz. One should notice that the

assumption Fz ̸= 0 for all (x, y) in the domain of z = z(x, y) isnecessary, which one can obtain explicitly if Fz is known.

Prepared by Dr. I.T. Leong for Tutorial 3-4 of Matb 210

Page 95: Tutorial Class III and IV 17-29/09/2012 1. First Order ...fstitl/Calculus2012/2013-example-03.pdf · Tutorial Class III and IV 17-29/09/2012.. 1.First Order Partial Derivatives; 2.Higher

. . . . . .

.

......

Example. Determine the extremum of the function z = z(x, y) definedimplicitly by the equation 3x2 + 2y2 + z2 + 8yz − z + 8 = 0.

Solution. Define F(x, y, z) = 3x2 + 2y2 + z2 + 8yz − z + 8, so thefunction z = z(x, y) is in fact the graph of the level surface Sassociated to the equation F(x, y, z) = 0, or sometimes we just denoteit by S : F(x, y, z) = 0. It follows that F(x, y, z(x, y)) = 0, for all (x, y) inthe (unspecified) domain of z(x, y), in fact we just think of the equalityas an identity in x and y.

So differentiate with respect to x and yrespectively by means of chain rule, we have

0 =∂

∂x(0) =

∂x( F(x, y, z(x, y)) ) =

∂F∂x

∂x∂x

+∂F∂z

∂z∂x

= Fx + Fz∂z∂x

, so

one has∂z∂x

(x, y) = −Fx(x, y, z(x, y))Fz(x, y, z(x, y))

= −Fx

Fzand

∂z∂y

(x, y) = −Fy(x, y, z(x, y))Fz(x, y, z(x, y))

= −Fy

Fz. One should notice that the

assumption Fz ̸= 0 for all (x, y) in the domain of z = z(x, y) isnecessary, which one can obtain explicitly if Fz is known.

Prepared by Dr. I.T. Leong for Tutorial 3-4 of Matb 210

Page 96: Tutorial Class III and IV 17-29/09/2012 1. First Order ...fstitl/Calculus2012/2013-example-03.pdf · Tutorial Class III and IV 17-29/09/2012.. 1.First Order Partial Derivatives; 2.Higher

. . . . . .

.

......

Example. Determine the extremum of the function z = z(x, y) definedimplicitly by the equation 3x2 + 2y2 + z2 + 8yz − z + 8 = 0.

Solution. Define F(x, y, z) = 3x2 + 2y2 + z2 + 8yz − z + 8, so thefunction z = z(x, y) is in fact the graph of the level surface Sassociated to the equation F(x, y, z) = 0, or sometimes we just denoteit by S : F(x, y, z) = 0. It follows that F(x, y, z(x, y)) = 0, for all (x, y) inthe (unspecified) domain of z(x, y), in fact we just think of the equalityas an identity in x and y. So differentiate with respect to x and yrespectively by means of chain rule, we have

0 =∂

∂x(0) =

∂x( F(x, y, z(x, y)) ) =

∂F∂x

∂x∂x

+∂F∂z

∂z∂x

= Fx + Fz∂z∂x

, so

one has∂z∂x

(x, y) = −Fx(x, y, z(x, y))Fz(x, y, z(x, y))

= −Fx

Fzand

∂z∂y

(x, y) = −Fy(x, y, z(x, y))Fz(x, y, z(x, y))

= −Fy

Fz. One should notice that the

assumption Fz ̸= 0 for all (x, y) in the domain of z = z(x, y) isnecessary, which one can obtain explicitly if Fz is known.

Prepared by Dr. I.T. Leong for Tutorial 3-4 of Matb 210

Page 97: Tutorial Class III and IV 17-29/09/2012 1. First Order ...fstitl/Calculus2012/2013-example-03.pdf · Tutorial Class III and IV 17-29/09/2012.. 1.First Order Partial Derivatives; 2.Higher

. . . . . .

.

......

Example. Determine the extremum of the function z = z(x, y) definedimplicitly by the equation 3x2 + 2y2 + z2 + 8yz − z + 8 = 0.

Solution. Let F(x, y, z) = 3x2 + 2y2 + z2 + 8yz − z + 8.

so∂z∂x

= −Fx

Fz= − 6x

2z + 8y − 1, and

∂z∂y

= −Fy

Fz= − 4y + 8z

2z + 8y − 1. To

locate the extremum value of z = z(x, y), one need its two partialderivatives zx and zy vanish, i.e. (6x, 4y + 8z) = (0, 0) where (x, y, z)is a point of the level surface S : F(x, y, z) = 0. Hence, x = 0, andy = −2z. Then 0 = F(0,−2z, z) = 2(−2z)2 + z2 + 8(−2z)z − z + 8 =

−7z2 − z + 8 = −(7z + 8)(z − 1) so z = 1 or − 87 . Hence P(0,−2, 1) or

Q(0, 167 ,− 8

7 ) are the only critical point of the function z = z(x, y),however, z = z(x, y) is not explicitly determined yet. One candetermine use the quadratic formula to express z in terms of x and y,and then one can see that zmax = 1 and zmin = − 8

7 .Remark. In the last part, we skip some details, but the gap can befilled in after we learn the second derivative test.

Prepared by Dr. I.T. Leong for Tutorial 3-4 of Matb 210

Page 98: Tutorial Class III and IV 17-29/09/2012 1. First Order ...fstitl/Calculus2012/2013-example-03.pdf · Tutorial Class III and IV 17-29/09/2012.. 1.First Order Partial Derivatives; 2.Higher

. . . . . .

.

......

Example. Determine the extremum of the function z = z(x, y) definedimplicitly by the equation 3x2 + 2y2 + z2 + 8yz − z + 8 = 0.

Solution. Let F(x, y, z) = 3x2 + 2y2 + z2 + 8yz − z + 8. so∂z∂x

= −Fx

Fz= − 6x

2z + 8y − 1, and

∂z∂y

= −Fy

Fz= − 4y + 8z

2z + 8y − 1.

To

locate the extremum value of z = z(x, y), one need its two partialderivatives zx and zy vanish, i.e. (6x, 4y + 8z) = (0, 0) where (x, y, z)is a point of the level surface S : F(x, y, z) = 0. Hence, x = 0, andy = −2z. Then 0 = F(0,−2z, z) = 2(−2z)2 + z2 + 8(−2z)z − z + 8 =

−7z2 − z + 8 = −(7z + 8)(z − 1) so z = 1 or − 87 . Hence P(0,−2, 1) or

Q(0, 167 ,− 8

7 ) are the only critical point of the function z = z(x, y),however, z = z(x, y) is not explicitly determined yet. One candetermine use the quadratic formula to express z in terms of x and y,and then one can see that zmax = 1 and zmin = − 8

7 .Remark. In the last part, we skip some details, but the gap can befilled in after we learn the second derivative test.

Prepared by Dr. I.T. Leong for Tutorial 3-4 of Matb 210

Page 99: Tutorial Class III and IV 17-29/09/2012 1. First Order ...fstitl/Calculus2012/2013-example-03.pdf · Tutorial Class III and IV 17-29/09/2012.. 1.First Order Partial Derivatives; 2.Higher

. . . . . .

.

......

Example. Determine the extremum of the function z = z(x, y) definedimplicitly by the equation 3x2 + 2y2 + z2 + 8yz − z + 8 = 0.

Solution. Let F(x, y, z) = 3x2 + 2y2 + z2 + 8yz − z + 8. so∂z∂x

= −Fx

Fz= − 6x

2z + 8y − 1, and

∂z∂y

= −Fy

Fz= − 4y + 8z

2z + 8y − 1. To

locate the extremum value of z = z(x, y), one need its two partialderivatives zx and zy vanish, i.e.

(6x, 4y + 8z) = (0, 0) where (x, y, z)is a point of the level surface S : F(x, y, z) = 0. Hence, x = 0, andy = −2z. Then 0 = F(0,−2z, z) = 2(−2z)2 + z2 + 8(−2z)z − z + 8 =

−7z2 − z + 8 = −(7z + 8)(z − 1) so z = 1 or − 87 . Hence P(0,−2, 1) or

Q(0, 167 ,− 8

7 ) are the only critical point of the function z = z(x, y),however, z = z(x, y) is not explicitly determined yet. One candetermine use the quadratic formula to express z in terms of x and y,and then one can see that zmax = 1 and zmin = − 8

7 .Remark. In the last part, we skip some details, but the gap can befilled in after we learn the second derivative test.

Prepared by Dr. I.T. Leong for Tutorial 3-4 of Matb 210

Page 100: Tutorial Class III and IV 17-29/09/2012 1. First Order ...fstitl/Calculus2012/2013-example-03.pdf · Tutorial Class III and IV 17-29/09/2012.. 1.First Order Partial Derivatives; 2.Higher

. . . . . .

.

......

Example. Determine the extremum of the function z = z(x, y) definedimplicitly by the equation 3x2 + 2y2 + z2 + 8yz − z + 8 = 0.

Solution. Let F(x, y, z) = 3x2 + 2y2 + z2 + 8yz − z + 8. so∂z∂x

= −Fx

Fz= − 6x

2z + 8y − 1, and

∂z∂y

= −Fy

Fz= − 4y + 8z

2z + 8y − 1. To

locate the extremum value of z = z(x, y), one need its two partialderivatives zx and zy vanish, i.e. (6x, 4y + 8z) = (0, 0) where (x, y, z)is a point of the level surface S : F(x, y, z) = 0. Hence, x = 0, andy = −2z.

Then 0 = F(0,−2z, z) = 2(−2z)2 + z2 + 8(−2z)z − z + 8 =

−7z2 − z + 8 = −(7z + 8)(z − 1) so z = 1 or − 87 . Hence P(0,−2, 1) or

Q(0, 167 ,− 8

7 ) are the only critical point of the function z = z(x, y),however, z = z(x, y) is not explicitly determined yet. One candetermine use the quadratic formula to express z in terms of x and y,and then one can see that zmax = 1 and zmin = − 8

7 .Remark. In the last part, we skip some details, but the gap can befilled in after we learn the second derivative test.

Prepared by Dr. I.T. Leong for Tutorial 3-4 of Matb 210

Page 101: Tutorial Class III and IV 17-29/09/2012 1. First Order ...fstitl/Calculus2012/2013-example-03.pdf · Tutorial Class III and IV 17-29/09/2012.. 1.First Order Partial Derivatives; 2.Higher

. . . . . .

.

......

Example. Determine the extremum of the function z = z(x, y) definedimplicitly by the equation 3x2 + 2y2 + z2 + 8yz − z + 8 = 0.

Solution. Let F(x, y, z) = 3x2 + 2y2 + z2 + 8yz − z + 8. so∂z∂x

= −Fx

Fz= − 6x

2z + 8y − 1, and

∂z∂y

= −Fy

Fz= − 4y + 8z

2z + 8y − 1. To

locate the extremum value of z = z(x, y), one need its two partialderivatives zx and zy vanish, i.e. (6x, 4y + 8z) = (0, 0) where (x, y, z)is a point of the level surface S : F(x, y, z) = 0. Hence, x = 0, andy = −2z. Then 0 = F(0,−2z, z) = 2(−2z)2 + z2 + 8(−2z)z − z + 8 =

−7z2 − z + 8 = −(7z + 8)(z − 1)

so z = 1 or − 87 . Hence P(0,−2, 1) or

Q(0, 167 ,− 8

7 ) are the only critical point of the function z = z(x, y),however, z = z(x, y) is not explicitly determined yet. One candetermine use the quadratic formula to express z in terms of x and y,and then one can see that zmax = 1 and zmin = − 8

7 .Remark. In the last part, we skip some details, but the gap can befilled in after we learn the second derivative test.

Prepared by Dr. I.T. Leong for Tutorial 3-4 of Matb 210

Page 102: Tutorial Class III and IV 17-29/09/2012 1. First Order ...fstitl/Calculus2012/2013-example-03.pdf · Tutorial Class III and IV 17-29/09/2012.. 1.First Order Partial Derivatives; 2.Higher

. . . . . .

.

......

Example. Determine the extremum of the function z = z(x, y) definedimplicitly by the equation 3x2 + 2y2 + z2 + 8yz − z + 8 = 0.

Solution. Let F(x, y, z) = 3x2 + 2y2 + z2 + 8yz − z + 8. so∂z∂x

= −Fx

Fz= − 6x

2z + 8y − 1, and

∂z∂y

= −Fy

Fz= − 4y + 8z

2z + 8y − 1. To

locate the extremum value of z = z(x, y), one need its two partialderivatives zx and zy vanish, i.e. (6x, 4y + 8z) = (0, 0) where (x, y, z)is a point of the level surface S : F(x, y, z) = 0. Hence, x = 0, andy = −2z. Then 0 = F(0,−2z, z) = 2(−2z)2 + z2 + 8(−2z)z − z + 8 =

−7z2 − z + 8 = −(7z + 8)(z − 1) so z = 1 or − 87 . Hence P(0,−2, 1) or

Q(0, 167 ,− 8

7 ) are the only critical point of the function z = z(x, y),however, z = z(x, y) is not explicitly determined yet. One candetermine use the quadratic formula to express z in terms of x and y,and then one can see that zmax = 1 and zmin = − 8

7 .Remark. In the last part, we skip some details, but the gap can befilled in after we learn the second derivative test.

Prepared by Dr. I.T. Leong for Tutorial 3-4 of Matb 210

Page 103: Tutorial Class III and IV 17-29/09/2012 1. First Order ...fstitl/Calculus2012/2013-example-03.pdf · Tutorial Class III and IV 17-29/09/2012.. 1.First Order Partial Derivatives; 2.Higher

. . . . . .

.

......

Example. Find second order partial derivative ∂2

∂y∂x z(x, y) of thefunction z = z(x, y) defined implicitly by the equation3x2 + 2y2 + z2 + 8yz − z + 8 = 0.

Solution. We follow the notation of the previous problem. In the last

problem, we have∂z∂x

= − 6x2z + 8y − 1

, and∂z∂y

= − 4y + 8z2z + 8y − 1

. So

we have zxy =∂zx

∂y=

∂y

(− 6x(2z + 8y − 1)2

)=

− 6x(2z + 8y − 1)2 · ∂

∂y(2z + 8y − 1) = − 6x

(2z + 8y − 1)2 · (2zx + 8) =

− 6x(2z + 8y − 1)2 · (8 − 12x

2z + 8y − 1).

Prepared by Dr. I.T. Leong for Tutorial 3-4 of Matb 210

Page 104: Tutorial Class III and IV 17-29/09/2012 1. First Order ...fstitl/Calculus2012/2013-example-03.pdf · Tutorial Class III and IV 17-29/09/2012.. 1.First Order Partial Derivatives; 2.Higher

. . . . . .

.

......

If the plane π : lx + my + nz = p and the quadric surfaceS : Ax2 + By2 + Cz2 = 1 are tangent to each other, determine therelationship among the coefficients.

Solution. The necessary and sufficient condition is l2A + m2

B + n2

C = p2.Note that both surfaces π and S are level surfaces defined byfunctions f (x, y, z) = lx + my + nz and g(x, y, z) = Ax2 + By2 + Cz2 Atthe common point P(x, y, z) (if exists) of two surfaces, both surfaceswill be tangent to each other, i.e. they have the same normal vectors,hence ∇f (x, y, z) = (l, m, n) and ∇g(x, y, z) = (2xA, 2yB, 2zC) areparallel, so (xA, yB, zC) = λ(l, m, n) for some non-zero λ, i.e.(x, y, z) = λ( l

A , mB , n

C ). As the coordinates of the common point

P(x, y, z) satisfy both defining equations, so p = λ(

l2A + m2

B + n2

C

)and

1 = λ2(

A l2A2 + B m2

B2 + C n2

C2

)= λ2

(l2A + m2

B + n2

C

)= λp, i.e.

p2 = p · p = pλ

(l2

A+

m2

B+

n2

C

)=

l2

A+

m2

B+

n2

C.

Prepared by Dr. I.T. Leong for Tutorial 3-4 of Matb 210

Page 105: Tutorial Class III and IV 17-29/09/2012 1. First Order ...fstitl/Calculus2012/2013-example-03.pdf · Tutorial Class III and IV 17-29/09/2012.. 1.First Order Partial Derivatives; 2.Higher

. . . . . .

.

......

If the plane π : lx + my + nz = p and the quadric surfaceS : Ax2 + By2 + Cz2 = 1 are tangent to each other, determine therelationship among the coefficients.

Solution. The necessary and sufficient condition is l2A + m2

B + n2

C = p2.On the other hand, suppose that the coefficients satisfy the equation,let P(x0, y0, z0) = P( l

Ap , mpB , n

Cp ), then

lx0 + my0 + nz0 =l2

Ap+

m2

pB+

n2

Cp=

1p

(l2

A+

m2

B+

n2

C

)=

p2

p= p,

i.e. P lies on π. And Ax20 + By2

0 + Cz20 = A l2

A2p2 + B m2

p2B2 + C n2

C2p2 =

1p2

(l2A + m2

B + n2

C

)= p2

p2 = 1, i.e. P lies on S too. So P lies on the

intersection of surfaces S and T. One then check ∇g = 2p∇f , and so

these are tangent to each at P.

Prepared by Dr. I.T. Leong for Tutorial 3-4 of Matb 210

Page 106: Tutorial Class III and IV 17-29/09/2012 1. First Order ...fstitl/Calculus2012/2013-example-03.pdf · Tutorial Class III and IV 17-29/09/2012.. 1.First Order Partial Derivatives; 2.Higher

. . . . . .

.

......

Example. Suppose that the variables P, V and T satisfy the equationof famous ideal gas law: PV = nRT, where n and R are constantsindependent of P, V and T. If P = P(V, T), V = V(P, T), andT = T(P, V) are implicit functions defined by means of the levelsurface S : PV = nRT.

(i) Evaluate the partial derivatives:∂P∂T

(T, V),∂T∂V

(P, V) and∂V∂P

(P, V).

(ii) Simplify∂P∂T

· ∂T∂V

· ∂V∂P

.

Solution. (i) As P = nRTV , so

∂P∂T

(T, V) =nRV

. Similarly T = PVnR , and

hence∂T∂V

(P, V) =P

nR. And V = nRT

P , so∂V∂P

(P, V) = −nRTP2 .

(ii)∂P∂T

· ∂T∂V

· ∂V∂P

= −nRV

· PnR

· .nRTP2 = −nRT

PV= −1.

Prepared by Dr. I.T. Leong for Tutorial 3-4 of Matb 210

Page 107: Tutorial Class III and IV 17-29/09/2012 1. First Order ...fstitl/Calculus2012/2013-example-03.pdf · Tutorial Class III and IV 17-29/09/2012.. 1.First Order Partial Derivatives; 2.Higher

. . . . . .

.Quiz 4 (14 minutes)..

......

...1 State the chain rule for the composite function of continuouslydifferentiable function f (x, y, z) and a differentiable curver(t) = (x(t), y(t), z(t)) in space.

...2 Suppose that x = r cos θ, y = r sin θ, and let z = f (x, y) be a twicecontinuously differentiable function. Defineg(r, θ) = f (r cos θ, , r sin θ). Express the partial derivatives:

(i) gθ(r, θ) =∂g∂θ

= , and

(ii)∂2g∂θ2 = ,

i.e. gθθ in terms of the partial derivatives of f ....3 Find the directional derivative Duf = of

f (x, y, z) =√

x2 + y2 − z2 at the point P(1, 1, 1) along the unitdirection u parallel to (1, 1,−1).

Prepared by Dr. I.T. Leong for Tutorial 3-4 of Matb 210